Сохранен 569
https://2ch.hk/sci/res/366963.html
24 декабря Архивач восстановлен после серьёзной аварии. К сожалению, значительная часть сохранённых изображений и видео была потеряна. Подробности случившегося. Мы призываем всех неравнодушных помочь нам с восстановлением утраченного контента!

МАТЕМАТИКА ДЛЯ НАЧИНАЮЩИХ, ТРЕД 8, ИДУЩИЙ ПЕРЕД ДЕВЯТЫМ

 Аноним 03/06/16 Птн 15:41:20 #1 №366963 
14649576805400.gif
14649576805401.jpg
14649576805402.jpg
14649576805403.jpg
ДЛЯ САМЫХ МАЛЕНЬКИХ:

Общие курсы
М. И. Сканави: "Элементарная математика".

Алгебра
И. М. Гельфанд, А. Шень: “Алгебра”. Весь курс школьной алгебры по 9 класс.
С. Б. Гашков: “Современная элементарная алгебра”.
Ю. М. Алимов, М. В. Колягин: "Алгебра и начала анализа".

Геометрия
Г. Коксетер: “Введение в геометрию“. Годная книга для уровня "продвинутый школьник".
А. Д. Александров, А. Л. Вернер, В. И. Рыжик: “Геометрия”. Учебник для 10-11 классов. Базовый и углубленный уровни.
Я. П. Понарин: “Элементарная геометрия” в двух томах. Собственно, первый том - это планиметрия, а второй том - это стереометрия.
А. Ю. Калинин, Д. А. Терешин: “Геометрия”, 10-11 классы. Годный учебник.

Тригонометрия
И. М. Гельфанд, С.М. Львовский, А. Л. Тоом: “Тригонометрия”. Название говорит само за себя. Много геометрических и физических интерпретаций + комплексные числа, как бонус.


БАЗОВЫЕ КУРСЫ ДЛЯ СТУДЕНТОВ:

Общая алгебра
Э. Б. Винберг: “Курс алгебры”. Пожалуй, лучший из известных учебников, соперничать с которым может разве что "Введение в алгебру" Кострикина.
А. И. Кострикин: “Введение в алгебру“. Пожалуй, лучший из известных учебников, соперничать с которым может разве что "Курс алгебры" Винберга.
М. Атья, И. Макдональд : "Введение в коммутативную алгебру".
А. Л. Городенцев: "Алгебра. Учебник для студентов-математиков". Вырос из лекций НМУ. Читать параллельно с Винбергом (Винберга читать в первую очередь).
И.Р. Шафаревич: “Основные понятия алгебры“. Замечательный обзор вообще того, что такое алгебра, как она выглядит и какое место она занимает в математике. Примеры, приложения и прочая конкретика.
E. Connell: Elements of Abstract and Linear Algebra". Хорошая первая книга по алгебре, да и математике вообще.
P. Grillet: "Abstract algebra". Очень лаконичный и понятный учебник. Надо знать элементарную теорию чисел, про индукцию, про множества и функции. Линейной алгебры нету.
J. Rotman: "Advanced modern algebra". Ротман сильно разжевывает. Задачи слишком простые для уровня учебника. Линейная алгебра есть.
M. Artin: "Algebra". Американский Винберг. Группы Ли, упор на геометрию. Задачи неудачные.
I. Herstein: “Topics in Algebra“. Прекрасные задачи, отбор материала очень устарел, почти что Ван дер Варден.
P. Aluffi: "Algebra, Chapter 0". Если ты в состоянии ее осилить, бери и забывай про остальные книжки из списка. Линейная алгебра есть.

Линейная алгебра
В. А. Ильин, Э. Г. Позняк: “Линейная алгебра“. Один из классических и самых популярных курсов линейной алгебры.
Д. В. Беклемишев: “Курс аналитической геометрии и линейной алгебры“.
И. М. Гельфанд: "Лекции по линейной алгебре". Не даётся определение определителя.
А. И. Кострикин, Ю. И. Манин: "Линейная алгебра и геометрия". Затрагивается темы геометрий и связей с квантовой механикой. Не даётся определение определителя.
S. Axler: "Linear algebra done right". Подход без определителей (почти). Одна из самых популярных книг за рубежом.
S. Treil: "Linear algebra done wrong". Не такая популярная, как Axler, но тоже хвалят, да. Определители есть.
G. Shilov: "Linear Algebra". Определитель появляется на первой странице.
K. Hoffman, R. Kunze: "Linear Algebra". Классика за рубежом.
P. Halmos: "Finite-Dimensional Vector Spaces". Тоже классика.
P. Peterson: "Linear Algebra". Не особо знаком, но выглядит аккуратно. Что-то вроде Акслера.
S. Roman: "Advanced Linear Algebra". Хороший учебник по линалу. Но нужно знать элементарные свойства матриц и определителей.

Математический анализ
T. Tao: “Real analysis“. Один из самых популярных курсов математического анализа на английском языке.
C. Pugh: "Real Mathematical analysis". Более простая версия Рудина с картинками. Норм книга, но не самая лёгкая.
У. Рудин: "Основы математического анализа".
В. А. Зорич: "Математический анализ". Первый том посвящен классическому анализу. Много примеров, много материала, в том числе даются в начале основы матлогики и теории множеств, а также функций между ними.
Р. Курант: "Курс дифференциального и интегрального исчисления". Идеален с точки зрения первого знакомства с теорией, но имеет достаточно сложные упражнения.
Г. М. Фихтенгольц: "Курс дифференциального и интегрального исчисления". Хорош как повторительный курс.
С. М. Львовский: "Лекции по математическому анализу". Записки лекций из НМУ. Нужно знать основы калькулюса.
Г. Г. Харди, Д. Е. Литтлвуд, Г. Пойа: "Неравенства".
Н. Н. Лебедев: "Специальные функции и их приложения".
Г. П. Толстов: “Ряды Фурье“.

Дифференциальные уравнения
С. Фарлоу: “Уравнения с частными производными для научных работников и инженеров“.

Вариационное исчисление
И. М. Гельфанд, С. В. Фомин: " Вариационное исчисление".

Топология
V. Runde: "A taste of topology". Неплохая книга по метрическим пространствам и общей топологии, затрагивает фундаментальную группу.
J. Strom: "Modern classical homotopy theory".
T. Dieck: "Algebraic topology".
M. Crossley: "Essential Topology". Пререквизит для изучения алгебраической топологии. Не затрагивает тему метрических пространств.


КУРСЫ ДЛЯ ПРОДВИНУТЫХ МАТЕМАТИКОВ

Математический анализ
А. И. Маркушевич: "Теория аналитических функций".
S. Ramanan: "Global calculus".
H. Amann, J. Echer: "Analysis".
W. Fidcher, I. Lieb: "A Course in Complex Analysis: From Basic Results to Advanced Topics".

Дифференциальные уравнения
В. И. Арнольд: “Обыкновенные дифференциальные уравнения”. Книга для уверенных в себе математиков. Диффеоморфизмы, фазовые потоки, гладкие многообразия. Слава Гермесу Трисмегисту!

Теория категорий
С. Маклейн: "Категории для работающего математика".
Р. Голдблатт: "Топосы. Категорный анализ логики".

Дифференциальная Геометрия
К. Номидзу: "Основы дифференциальной геометрии".
J. Lee: "Manifolds and DIfferential Geometry".
L. Nicolaescu: "Lectures on the Geometry".
P. Michor "Topics in Differential Geometry".

Алгебраическая геометрия
Д. Мамфорд: "Красная книга о многообразиях и схемах".
В. В. Острик, М. А. Цфасман: “Алгебраическая геометрия и теория чисел: рациональные и эллиптические кривые”.
В. И. Арнольд: “Вещественная алгебраическая геометрия”.
Ю. И. Манин: Введение в теорию схем и квантовые группы“.
R. Vakil: "Foundations of algebraic geometry".
S. Bosch: "Algebraic Geometry and Commutative Algebra".
U. Gotz, T. "Wedhorn: Algebraic Geometry".
E. Harris: "The Geometry of Schemes".

Топология
А. Хэтчер: "Алгебраическая топология".
J. Munkres: "Topology". Книга - жесткий учебник по теоретико-множественной топологии. Много ненужного для других областей математики.


ИНТЕРЕСНОЕ:

Цикл “Manga guide to...“. Популярное изложение различных областей математики (и не только), оформленное в виде манги. Увы, без фансервиса.
Н. А. Вавилов: “Конкретная теория групп I: основные понятия“. И вообще все остальные книги (и лекции!) Вавилова.
П. С. Александров: “Введение в теорию групп“. Просто о сложном. Несколько вольный язык изложения, местами затрудняющий восприятие.
В. Б. Алексеев: “Теорема Абеля в задачах и решениях”.
Р. Курант, Г. Роббинс: “Что такое математика?”. Очень интересная книга, в двух словах не описать. Но вас захватит, надолго.
Н. Я. Виленкин: "Рассказы о множествах". Теория множеств для широкого круга читателей.
М. М. Постников: “Теорема Ферма. Введение в теорию алгебраических чисел”.
Н. Стинрод: “Первые понятия топологии“.
А. Я. Хинчин: “Три жемчужины теории чисел“.
О. Я. Виро, О. А. Иванов, Н. Ю. Нецветаев, В. М. Харламов: “Элементарная топология”.
Я. П. Понарин: “Алгебра комплексных чисел в геометрических задачах”.
А. А. Заславский: “Геометрические преобразования”.
В. Акопян, А. А. Заславский: “Геометрические свойства кривых второго порядка”.
В. И. Арнольд: “Геометрия комплексных чисел, кватернионов и спинов”.
В. В. Прасолов: “Геометрия Лобачевского”.
Д. В. Аносов: “Дифференциальные уравнения: то решаем, то рисуем”.
В. В. Прасолов: “Наглядная топология”.
Д. В. Аносов: “От Ньютона к Кеплеру”.
М. Клайн: “Математика. Поиск истины“.
Д. Пойа: “Математическое открытие“.
Л. Кэрролл: “Логическая игра“.
Д. Пойа: “Как решать задачу“.
О. Я. Виро, Д. Б. Фукс: "Введение в теорию гомотопий. Гомологии и когомологии".
A. Ostermann, G. Wanner: "Geometry by its history".
T. Sundstrom: "Mathematical reasoning writing and proof". В книге объясняется что такое математическое доказательство, математический факт и каким образом их можно придумывать. Начала теории множеств.
D. Dummit R. Foote: “Abstract Algebra“. Много примеров, задач, но страшно скучный учебник, его нужно держать как справочник.


ПОЛЕЗНЫЕ РЕСУРСЫ:

Библиотечка "Квант": math.ru/lib/ser/bmkvant
Высшая математика просто и доступно, по 2 курс включительно: mathprofi.net
Необъятная онлайн библиотека: gen.lib.rus.ec
Аноним 03/06/16 Птн 16:05:53 #2 №366975 DELETED
Нахуй нужны математики в XXI веке, если есть компьютеры? Чем вообще занимаются математики в своих нынешних исследованиях?
Настраивают компьютер, который за них всё делает?
Аноним 03/06/16 Птн 16:08:39 #3 №366976 DELETED
>>366975
Мамку твое ебут. Иди на хуй теперь, пидор.
Аноним 03/06/16 Птн 16:21:39 #4 №366981 DELETED
Гоните этого дебила >>366975 насмехайтесь над ним. Малолетний уебок и правда думает что математика это то что он в школе не может осилить.
Аноним 03/06/16 Птн 16:24:15 #5 №366982 DELETED
>>366975
Математики незаменимы. Это самый эффективный метод добычи энергии. Собрал математиков, поставил жопой к котлу, показал математикам физиков, химиков и особенно биологов, и от раскаленного бугурта котел превращается в турбореактивный двигатель, способный столкнуть Землю с ее родной орбиты.
Аноним 03/06/16 Птн 16:40:58 #6 №366986 DELETED
>>366982
Физика — это такой раздел математики, в котором эксперименты дорогие.
Аноним 03/06/16 Птн 16:45:38 #7 №366988 
Ебанутся. С такими тредами по математике надо создавать еще один тред, математика для нематиматиков. Бог любит троицу.
Аноним 03/06/16 Птн 16:49:41 #8 №366990 
>>366988
Пойду составлю списочек пока. Для этого придется читать и читать, но думаю к десятому треду создам отдельный, с НОРМАЛЬНЫМ спискомлитературы, а не говном, которое мало кто кроме писателя и редактора читал.
Аноним 03/06/16 Птн 16:58:12 #9 №367000 
>>366990
>>366988
Ну вы и ретарды, всё равно вся активность всегда только в одном треде.
Аноним 03/06/16 Птн 17:07:00 #10 №367004 DELETED
>>366986
>Физика — это такой раздел математики, в котором эксперименты дорогие.
И в отличии от математики, эксперименты в физике есть
И полезные
И интересные
И физику
И журнашлюхе
И имеют отношение к реальности
И являются основой естественных наук и познания окружающего мира
Аноним 03/06/16 Птн 17:50:07 #11 №367023 DELETED
>>367004
>И в отличии от математики, эксперименты в физике есть
В отличие от математики, в физике роль экспериментов гипертрофирована и сделана предметом культа, чего нет в других разделах математики. Фактически, физика — это маленькая математическая секточка радикальных экспериментаторов, характеризующаяся усиленным выбиванием финансирования и агрессивным пиаром.
>И физику
>И журнашлюхе
То есть пристрастному автору и безграмотному обывателю.
>И имеют отношение к реальности
>И являются основой естественных наук и познания окружающего мира
Это касается всей математики, а не только её кусочка под названием „физика“.
Аноним 03/06/16 Птн 17:54:32 #12 №367029 DELETED
>>366981
Пошла на хуй, Ира.
Аноним 03/06/16 Птн 17:57:55 #13 №367031 
>>367000
> активность
> 90% постов - уёбы спрашивающие А ПАСАВЕТУЙТЕ КНИЖКУ))
Лучше меньше да лучше.
Аноним 03/06/16 Птн 18:06:27 #14 №367036 
>>367031
Ох, лол, шел бы ты отсюда с таким мировоззрением, петушок. Это тред для ПАСАВЕТУЙТЕ КНИЖКУ как раз и создан.
Аноним 03/06/16 Птн 18:16:41 #15 №367038 DELETED
>>367023
>То есть пристрастному автору и безграмотному обывателю.
Отсюда и признание, и богатства, и уважение, лол.
>Это касается всей математики
Математики это в принципе не касается. Все что она способна дать - молоток для физика, который сможет им гвоздь забить и дом построить. Покажи математику самый примитивный линейный дифур и он физическое решение от нефизического отличить не сможет, потому что для математика разницы не видит (и не должна). 99% мат. конструкций - абстракции противоречащие реальности, но видать на мехматах предпочитают о таких деталях умалчивать. Основа естественных наук - эксперимент. Во всяком случае пока с этим согласен весь мир, кроме обиженных математиков. Даже всякие социопсихолухи это понимают и пытаются подстроиться под реалии.
Аноним 03/06/16 Птн 18:33:32 #16 №367043 DELETED
>>367038
Почему в треде о математике говорят про естественные науки? Своих тредов что-ли нет?
Аноним 03/06/16 Птн 18:36:48 #17 №367045 DELETED
>>367038
Ох уже этот неокантизм, вроде в 21 веке, а мировоззрение как в 19.
Аноним 03/06/16 Птн 18:44:39 #18 №367048 
>>367000
Ну и похуй. Зато будет нормальная активность, без этого всего типа ЗА ЧИСТОТУ МАТЕМАТИКИ ЗИГ ХАЙЛЬ ZFC или ВСЕХ ФИЗИКОВ НАДО УНИЧТОЖИТЬ!! ОНИ РАК УБИВАЮЩИЙ МАТЕМАТИКУ
Аноним 03/06/16 Птн 18:46:57 #19 №367052 DELETED
>>367043
Простите, я больше не буду. Мне просто показалось, что кто-то тут математику наукой считает. Рад знать что ошибался.
Спасибо.
Аноним 03/06/16 Птн 18:51:06 #20 №367057 DELETED
>>367045
Ты что, не добрал свою дозу гомотопического хаоса?
Аноним 03/06/16 Птн 18:56:13 #21 №367059 
>>366988
Так это и есть такой тред, одно унылое школьное говно в списке.
Аноним 03/06/16 Птн 19:03:17 #22 №367061 
>>367059
>алгебраическая топология
>школьное говно
Аноним 03/06/16 Птн 19:04:09 #23 №367062 
>>367061
>никто не может решить сколько будет 2^2^2 без гугла
Аноним 03/06/16 Птн 19:05:06 #24 №367063 DELETED
>>367038
Опять пошла знакомая шарманка.
КРИТЕРИИ «НАУЧНОСТИ» ВРОДЕ КРИТЕРИЯ ПОППЕРА БЫЛИ СПРОЕКТИРОВАНЫ С ОДНОЙ ЕДИНСТВЕННОЙ ЦЕЛЬЮ — ОТДЕЛИТЬ ФИЗИКУ ОТ МАТЕМАТИКИ, А ЭТО В СВОЮ ОЧЕРЕДЬ НУЖНО ИСКЛЮЧИТЕЛЬНО ДЛЯ ПРИВИЛЕГИРОВАННОГО СТАТУСА И ФИНАНСИРОВАНИЯ.
Абсурдный миф о «практичности» физики (в противоположность «оторванности от реальности» математики) усиленно (и совершенно искусственно!) внедряется в разум школьников и студентов тоталитарной сектой физиков (математики-отступники), чтобы компенсировать (справедливо возникающее и абсолютно верное!) мнение о большей фундаментальности и приоритете математики над физикой, подтверждаемое, например, тем, что математика начинает преподаваться гораздо раньше (и, как правило, гораздо лучше!) этого своего частного приложения.
Математики же, ввиду своей природной скромности, соглашаются с гегемоном «физиков» и постоянным ущемлением своей значимости в «науке» (ведь фактически НАУКА И МАТЕМАТИКА ЭТО СИНОНИМЫ).
Про «ЭКСПЕРИМЕНТ» отдельный разговор.
Аноним 03/06/16 Птн 19:06:15 #25 №367064 
>>367061
Я же не подразумевал своей фразой буквализм "в списке все школьное говно", у меня типичная гипербола, означающая, что слишком много материала для школьника (т.е. ограниченного в области школы, не имеющего дальнейшего развития; или составляющего долгое топтание на месте), хотя это /sci, а не /un.
Аноним 03/06/16 Птн 19:10:22 #26 №367065 DELETED
>>367063
>ЛЮДИ ДОБРЫЕ, ВЫ ПОСМОТРИТЕ ЧТО ДЕЛАЕТСЯ!!! ФИЗИКИ МАТЕМАТИКУ УНИЖАЮТ!!!!!!! ГОВОРЯТ ЧТО ОНА АБСТРАКТНАЯ!!! А ОНА НЕ АБСТРАКТНАЯ!!! ОНА САМАЯ ЧТО НИ НА ЕСТЬ НАСТОЯЩАЯ!! ГОНИТЕ ФИЗИКОВ, НАСМЕХАЙТЕСЬ НАД НИМИ!!! СЛАВА МАТЕМАТИКЕ, УБИТЬ ВСЕХ ФИЗИКОВ, РЯЯЯЯЯЯЯ
Аноним 03/06/16 Птн 19:13:49 #27 №367066 DELETED
>>367063
ТЕЗИС ДЮЭМА-КУАЙНА БЫЛ СПРОЕКТИРОВАН С ОДНОЙ ЕДИНСТВЕННОЙ ЦЕЛЬЮ — ДИСКРЕДИТИРОВАТЬ ФИЗИКУ КАК ЕДИНСТВЕННУЮ ЕСТЕСТВЕННУЮ НАУКУ
Аноним 03/06/16 Птн 19:14:07 #28 №367067 
>>367062
Так тут начинающие. По-твоему, они сразу должны пространствами тейхмюллера оперировать? У всех начальный уровень разный. Считать умеют - уже хорошо.

>слишком много материала для школьника
Например? Для школьников 9 книг. Все остальное выходит за рамки необходимого не интересующимся математикой.
Аноним 03/06/16 Птн 19:15:16 #29 №367068 
>>367067
Из этих 9 книг, дай бог чтобы 3 были полезные.
Аноним 03/06/16 Птн 19:17:09 #30 №367069 
>>367068
Ну е-мое. Почему в этом треде всегда проблемы с осознанием факта, что разные люди воспринимают информацию по-разному? То, что тебе кажется говном кому-то может быть самым понятным изложением.
Аноним 03/06/16 Птн 19:19:00 #31 №367070 DELETED
>>367065
Всё, что до этого предложения:
>СЛАВА МАТЕМАТИКЕ, УБИТЬ ВСЕХ ФИЗИКОВ, РЯЯЯЯЯЯЯ
— правда. Никого убивать не надо, необходимо восстановить баланс и открыть людям глаза на организованную промывку мозгов так называемыми «физиками».
Аноним 03/06/16 Птн 19:19:41 #32 №367071 
>>367069
Ты ебанулся? Хотя чего я спрашиваю это у математика...
То что они написанны разными людьми не значит что они отличаются друг от дружки. Там все описывается одинаково. Если тебе не понятно что то то надо перечитать еще раз. Или спросить в этом треде, в котором тебя по привычке смешают с говном и начнут задвигать за чистоту арийской рассы математики
Аноним 03/06/16 Птн 19:20:03 #33 №367072 DELETED
>>367070
А еще толще можешь?
Аноним 03/06/16 Птн 19:21:32 #34 №367073 
>>367071
Зачем троллишь? К тому, же я не математик.
Аноним 03/06/16 Птн 19:21:48 #35 №367074 DELETED
>>367072
Толстая мамаша твоя, а я абсолютно серьёзен. Впрочем, пойти в отрицалово и обвинения в троллинге — тоже выход.
Аноним 03/06/16 Птн 19:22:59 #36 №367075 DELETED
>>367074
>Всё, что до этого предложения:
>СЛАВА МАТЕМАТИКЕ, УБИТЬ ВСЕХ ФИЗИКОВ, РЯЯЯЯЯЯЯ
>— правда.

>математика не абстрактна

Ну давай, найди мне число два во вселенной.
Аноним 03/06/16 Птн 19:23:47 #37 №367076 
>>367073
Я правду говорю. Я тут когда спрашивал про книги меня говном закидали, а потом оправдались "да у меня прост в жизни проблемы) че ты репортишь то?"
Аноним 03/06/16 Птн 19:24:33 #38 №367077 DELETED
>>367075
Сперва ты найди мне во вселенной дом.
Аноним 03/06/16 Птн 19:25:25 #39 №367078 
>>367076
Ну мне тут много раз поясняли. Правда специфическим языком, но с символьной разметкой и, вроде бы, от души.
Аноним 03/06/16 Птн 19:25:40 #40 №367079 
>>367067
По мне, начинающие никогда не дойдут до алгеома, если будут действовать по списку, а именно ебашить школьную геометрию (имхо вообще бесполезную, которую можно вычеркнуть, потеряв при этом нихуя), школьную алгебру (разве что для развития навыка записи в тетрадочке, но не на таком же блевотно-однотипном материале, гельфанд шень это выше крыши); при этом не получат никакой общематематической культуры и самого важного навыка: доказательства.
После этого начинать Винберга, Рудина, Кострикина-Манина, это будет как удар лбом об стену. Потому что надо доказывать, надо теоретизировать, а родных циферок и выводов из евклида нету.
Аноним 03/06/16 Птн 19:26:34 #41 №367080 DELETED
>>367077
И это все что ты можешь? Я ожидал от маняматика чего то большего.
Дом это место в котором человек считает себя защищенным от сил мира с удобствами для комфортной жизни.
Аноним 03/06/16 Птн 19:30:12 #42 №367081 DELETED
>>367080
Я не просил тебя описать дом, я просил тебя ПОКАЗАТЬ дом. Описывать можно и (якобы) „абстрактные“ математические понятия, с этим даже конченая физикоблядь спорить не станет.
Аноним 03/06/16 Птн 19:32:54 #43 №367082 DELETED
>>367081
>Сперва ты найди мне
>я просил тебя ПОКАЗАТЬ
Очень плохая логика у тебя. Я думал намного лучше будет.
Аноним 03/06/16 Птн 19:34:12 #44 №367083 DELETED
>>367080
>алгеома
Алгебраическая геометрия изучается и активнейшим способом используется в школе.
Аноним 03/06/16 Птн 19:34:15 #45 №367084 DELETED
>>367081
Ебать какой софист нашелся. Сперва определи, что ты подразумеваешь под "показать" и под "дом", иначе ты сможешь вилять жопой в своих терминах как танцор гоу-гоу.
Аноним 03/06/16 Птн 19:35:19 #46 №367085 DELETED
>>367083
Мне представляется, что заходя в этот тред, люди хотят стремится к лучшему, к продвинутой математике. А если человеку надо ебануть школьных учебников, он идет в раздел /un и просит там совета.
Аноним 03/06/16 Птн 19:35:34 #47 №367086 DELETED
>>367084
>Go-go танцевали не только девушки, но и мужчины, особенно это было характерно в 60-х годах для гей-клубов.
Аноним 03/06/16 Птн 19:36:37 #48 №367087 DELETED
>>367082
Тогда это — двойка.
Аноним 03/06/16 Птн 19:36:42 #49 №367088 DELETED
>>367086
У нас тут оказывается гей клуб.
Аноним 03/06/16 Птн 19:37:02 #50 №367089 DELETED
>>367087
Нет, это два бокала всего лишь.
Аноним 03/06/16 Птн 19:42:56 #51 №367090 DELETED
>>367089
Это манифестация идеи двойки, так же как на твоём изображении не дом, а манифестация идеи дома. На понятие «дом» указать невозможно, можно указать на конкретный дом, точно так же на понятие «двойка» указать невозможно, но я показал конкретную двойку.
Разницы никакой нет. Разница вероломно навязывается безответным школьникам, а потом (когда вырастут) они не понимают, как можно думать иначе.
Аноним 03/06/16 Птн 19:49:06 #52 №367093 DELETED
>>367090
Платонист в треде, все в мир идей.
Аноним 03/06/16 Птн 19:50:24 #53 №367095 DELETED
>>367093
Блядь, только хотел написать про мир идей, а уже написали!
Аноним 03/06/16 Птн 19:57:23 #54 №367096 DELETED
>>367090
Ясно.
А я перед этим долбоебом рапинался.
Аноним 03/06/16 Птн 19:59:00 #55 №367097 
Итак, что будем делать с нитью? Надо её модернизировать/реабилитировать.
Аноним 03/06/16 Птн 20:02:10 #56 №367099 DELETED
>>367090
А как манифестирует себя идея бесконечности или иррациональности? А манифестация множеств {дом1,дом2} и {{дом1},{дом2}} отличается или нет?
Аноним 03/06/16 Птн 20:03:53 #57 №367101 
ЗАТКНИТЕСЬ НАХУЙ
Аноним 03/06/16 Птн 20:06:56 #58 №367102 
>>367097
Нужен 2ch.hk/math.
Подобно тому, как /ftb вырос в отдельный раздел из /sp, математика уже не вмещается в /sci. Половина мат. тредов - срачи между истинными хозяевами треда - математиками - и грязными физиками, сующими свой грязный нос-обрубок туда, куда не следует.
Ноудискасс.
Аноним 03/06/16 Птн 20:12:26 #59 №367103 
Так давайте, пишите-звоните, щелкайте обезьяну по носу.
Аноним 03/06/16 Птн 20:14:25 #60 №367104 DELETED
>>367084
Ему-то зачем это определять, это не он утверждает то что кок пок кок математика эта абстракция, а всё астальное типа нет.
Аноним 03/06/16 Птн 20:15:28 #61 №367105 
>>367079
Кому-то и школьный курс освоить по каким-либо соображениям надо. Да и не подразумевает список, что по нему от начала до конца доходить надо. Разумеется, какую-нибудь теорию гомотопий освоить самостоятельно крайне тяжело. Но это не проблема списка, а проблема сложности и контринтуитивности некоторых разделов математики. Помочь тут может разве что наставник, разбирающийся в теме, который и подскажет и исправит и задач задаст порешать. А в остальном по этому списку вполне реально учиться математике. Я смог в Винберга с Тао вкатиться с, можно сказать, школьными знаниями (учился в языковой школе, поэтому с математикой было плохо). Не такое уж достижение, но уже что-то. Единственное что, я бы листки какого-нибудь сунца в школьный список добавил. Но они же вроде без названий.

>>367099
Обсуждать. Вежливо, тактично, аргументированно. Без "выкинуть все нахуй, и оставить только вот эти три книги". Без срача "Вербицкий против всех" или "Дифференциальная геометрия не нужна".
Аноним 03/06/16 Птн 20:18:23 #62 №367106 
>>367103
Мразь молчала два месяца назад. Попробуйте сейчас кто-нибудь.
Аноним 03/06/16 Птн 20:18:59 #63 №367107 
>>367103
>>367102
Не вижу, чем тред не справляется. По-моему слишком низкая активность для доски.
Аноним 03/06/16 Птн 20:20:36 #64 №367108 
>>367105
>они же вроде без названий
Да ну?
http://www.mccme.ru/free-books/

Тем не менее большинству полезно освежить школьные знания, дополнить их продвинутым уровнем. А школьный учебник на 80% состоит из воды и жевания. А самое главное, не учит конструктивности, там все построено на обучению каким-то конкретным методам.
Аноним 03/06/16 Птн 20:27:02 #65 №367110 
>>367107
Соглашусь. Тут вон даже обычный тред математики неактивным стал. На целую доску не потянет.

>>367108
>Да ну?
Ну ок, че. Мне их всегда просто отдельными листками кидали.

>Тем не менее большинству полезно освежить школьные знания, дополнить их продвинутым уровнем. А школьный учебник на 80% состоит из воды и жевания. А самое главное, не учит конструктивности, там все построено на обучению каким-то конкретным методам.
Ни одного плохого школьного учебника в списке нет. Да, подход изложения не везде с заделом на профессиональную карьеру математика. Но и не всем это нужно.
Аноним 03/06/16 Птн 20:32:15 #66 №367111 
>>367110
Даже хороший школьный учебник обязан:
1) быть распределенным на определенное количество часов
2) охватывать все темы для егэ
Так что в итоге оказывается растянутым, с большим количеством лишней инфы.
Аноним 03/06/16 Птн 20:32:17 #67 №367112 
Израиль Моисеевич совершенно потрясающе комментировал доклады. Он не разрешал комментарии со стороны аудитории, иногда вопросы задавались, но это И.М. не очень приветствовалось. Более того, он очень часто запрещал какие-либо комментарии и со стороны докладчика. Докладчик, если он был математиком (для физиков делалось исключение), должен был докладывать по такому принципу: определение, теорема, лемма, доказательство. Ни в коем случае докладчик не должен был сам рассказывать, зачем это нужно, почему это интересно. Это было полностью прерогативой И.М. и делал он это блистательно, а иногда чрезвычайно зло.

У него было несколько коллег и учеников, которых он постоянно оскорблял во время их докладов. Они были милейшие люди и это терпели. Причем оскорбления были явными, а не только «это – глупость» или «ты делаешь не то, что нужно». Мне запомнился такой яркий эпизод, на котором я присутствовал, и уверен, что он был не единственным в этом роде.

На семинаре Гельфанда делал доклад известный математик средних лет по своей тематике. Давайте назовем его профессором Ивановым. Иванов начал выступать, пару раз его перебивал Гельфанд, а где-то минут через 15 Гельфанд остановил речь Иванова и заявил, что тот подход, который предлагает профессор Иванов, совершенно никуда не годится, что это глупость, что здесь нужно делать не так, а нужно делать совершенно по-другому. Дальше Израиль Моисеевич обратился к одному из школьников и сказал. Назовем школьника Мишей. «Миша, Вам понятно то, что я сейчас объяснил?» Мише это было понятно, тогда И.М. стал объяснять дальше, как это нужно делать и сказал следующее. «Продолжать слушать этот доклад мы не будем, это совершенно бессмысленно, перейдем к следующему докладу. А на следующем заседании семинара Миша расскажет, как нужно на самом деле расправляться с этой тематикой».

И действительно, на следующем заседании Миша, который впоследствии стал очень известным математиком, рассказал, как это нужно делать. Я уверен, хотя не был этому свидетелем, что школьник Миша приезжал к Гельфанду домой, они там вместе поработали и разобрались в проблеме. Однако внешне это выглядело таким образом, что бедному Иванову была публично нанесена пощечина, объяснено, что он ничего не понимает в своей собственной области, что он не знал, как делать правильно, а вот школьник Миша может это сделать, как следует.
Аноним 03/06/16 Птн 20:41:00 #68 №367114 
>>367111
Не вижу ничего плохого в том, чтобы научиться решать уравнения с параметрами и упрощать выражения. Или что там еще сейчас в ЕГЭ. В крайнем случае можно что-нибудь пропустить. В реал анилизисе Тао тоже много воды, однако это не минус, а плюс. Так иногда понятнее.

>>367112
Это откуда?
Аноним 03/06/16 Птн 20:47:46 #69 №367116 
Этот матанопетушок никак не успокоится. Неужели это N даун так мутировал?
Еще и забывает кто маняматику двигает а кто дрочит ZFC теории.
Аноним 03/06/16 Птн 20:56:43 #70 №367117 
>>367114
Давай более наглядно, в Алимове из оп-списка 114 страниц посвящено тригонометрии, в них содержатся 375 упражнений.
Охватываемые темы: определения основных функций, их свойства, основные тождества и разнообразные выводы из них.
Охуенно, да? Это, конечно, не учебник геометрии, где на протяжении 1000 упражнений неуверенно разрабатывается аксиоматика Евклида, но тоже достойно.
Аноним 03/06/16 Птн 20:59:45 #71 №367118 
>>367112
drgx, тебе же Каледин все сказал уже.
Аноним 03/06/16 Птн 21:03:04 #72 №367119 DELETED
>>366982
>>367004
>риальность кокок риальность не изучаешь риальный мир не мужык
Как будто математику не похер на прикладникопроблемы. Математика изучает мир, порожденный только законами логики, а не законами логики плюс кучкой рандомных уравнений с рандомными константами. Что сразу говорит о том, что математика смотрит шире. Если завтра в нашем мирке изменится какая-нибудь там постоянная Планка, математике не сделается ничего, а половина трудов по физике отправится на помойку. А если правило modus ponens перестанет выполняться, то нахуй пойдут вообще все, и всякие физики в первую очередь, потому что ни один эксперимент не будет доказывать вообще ничего, даже то, что второй раз этот же эксперимент даст то же самое, даже при условии верности и достаточности описывающей его теории.
Аноним 03/06/16 Птн 21:08:42 #73 №367120 
Люди делятся на три типа:
• логикофанатики
• физикомафиозы
• платонобоги
Аноним 03/06/16 Птн 21:09:55 #74 №367121 
>>367117
То есть тебе не нравится, что много упражнений и мало тем? По мне так лучше уж это, чем наоборот.
Аноним 03/06/16 Птн 21:27:52 #75 №367122 
это норм, что не могу понять такие вещи? см. ниже доказательство.
что я сделал. я нашел в computer algebra обратную матрицу простой матрицы 2на2. потом построил, как я понял, из объяснения ниже, столбец матрицы Х. решил ее в computer algebra и сравнил с столбцом обратной матрицы.
но это фигня какая-то, учитывая что из доказательсва я должен понять как находить обратные матрицы.

мне бы хотелось книгу где понятнее.

это из Винберг - курс алгебры.

Теорема 7. Квадратная матрица обратима тогда и только тогда, когда она невырожденна.

Доказательство.
...
Обратно, пусть матрица А невырожденна. Будем искать
обратную матрицу как решение матричного уравнения АХ = Е. Для
элементов k-ro столбца матрицы X это дает систему линейных
уравнений с матрицей коэффициентов А и столбцом свободных членов,
равным k-му столбцу матрицы Е.


Аноним 03/06/16 Птн 21:31:33 #76 №367124 
>>367122
Можно только say, что ты extremely bad умеешь обращаться с computer algebra, потому что, если бы ты всё делал correctly, то получил бы right answer.
Аноним 03/06/16 Птн 21:32:45 #77 №367125 
>>367122
Зачем тут компьютер, нужно просто понимать, что такое произведение матриц.
Аноним 03/06/16 Птн 21:32:45 #78 №367126 
>>367122
Когда ты строишь обратную матрицу, то делишь каждый элемент транспонированной матрицы алгебраических дополнений на определитель исходной матрицы. У вырожденной матрицы определитель равен нулю, поэтому нельзя построить ее обратную. Или ты не про это?
Аноним 03/06/16 Птн 21:39:12 #79 №367128 DELETED
>>367119
А я возьму из математики исключу закон исключенного третьего, сразу придется все передоказывать!
Аноним 03/06/16 Птн 21:41:30 #80 №367129 
>>367121
Упражнений просто ПИЗДЕЦ много, на такую малополезную и необъемную тему. А уж содержание этих упражнений... Почти все посвящены подставлению тождеств и прочим комбинаторным говном.
Аноним 03/06/16 Птн 21:43:58 #81 №367130 
>>367129
Не нужно, кстати, недооценивать простые упражнения на использование формул. Для меня, в свое время, тождественные преобразования были понс асинорум для понимания алгебры вообще.
Аноним 03/06/16 Птн 21:51:12 #82 №367131 
14649798724430.png
>>367130
На кой это надо вообще? Система компьютерной алгебры все равно знает больше трюков (чем пикрелейтед) и не совершает ошибок.
Аноним 03/06/16 Птн 21:52:44 #83 №367132 
>>367131
Я не вижу что на картинке.
Аноним 03/06/16 Птн 21:53:58 #84 №367134 
>>367132
Шпора по тригонометрии.
Аноним 03/06/16 Птн 22:38:59 #85 №367138 
>>367125
>>367126
>>367124

а понятно то, что я выделил курсивом в доказательстве?
Аноним 03/06/16 Птн 22:42:33 #86 №367139 
>>367138
Понятно.
Аноним 03/06/16 Птн 22:51:15 #87 №367141 
>>367118
che
Аноним 03/06/16 Птн 22:53:14 #88 №367142 
>>367124 а вот и русикобыдло подьехало 1 газоочистителя то уже накатил долбоёб? за ватную рашку свою уже путину помолоился?
Аноним 03/06/16 Птн 22:56:33 #89 №367144 
>>367142
Я think, что у вас butthurt, советую do a barell roll, и suck a dick. Я вообще мамкин русофоб, нахуй иди.
Аноним 03/06/16 Птн 23:36:32 #90 №367150 
>>367138
А что тут может быть непонятно? Есть вектор-столбец переменных из X, при умножении строк-коэффициентов из A на него получается система уравнений. Уравнений столько, сколько строк. Переменных в каждом уравнении столько, сколько элементов в столбце из X.
Аноним 04/06/16 Суб 05:33:24 #91 №367153 
Сап, математичи!
Недавно возникло ощущение, что я просёк "фишку" математики. Хотя никаких глубоких знаний у меня нет - обычный университетский курс матан+линал+диффуры(включая урматы)+теорвер+стохи+вычислялка+тфкп. Сдавал все на 3-4, теоремы сам не доказывал, но таки интересовался и пытался все связать воедино.
Стоит ли далее углубляться в математику, если сфера моих интересов - межмолекулярные взаимодействия, нелинейные эффекты на таких масштабах (0.1-100нм), моделирование систем типа белок-белок-раствор?
Какие могут еще ждать за поворотом откровения? Типа дуализма элементов множеств и операторов, типа базиса, типа линейных пространств, типа колец и алгебр и типа фурье и типа банаховых пространств и групп ли? Какие еще есть ништяковые идеи в математике, которые пригодятся прикладнику? Спасибо.
Аноним 04/06/16 Суб 07:35:45 #92 №367154 
>>367153
> прошел кучу университетских курсов
> базисы, линейные пространства, кольца, алгебры называет откровениями
Вся суть университетского образования.
Аноним 04/06/16 Суб 08:30:34 #93 №367157 DELETED
>>367119
Проиграл с фантазий лингвиста.
Аноним 04/06/16 Суб 10:05:15 #94 №367162 
Сука ну, Два с половиной школьникостудента, один сверхмассивный толстяк и полтора адекВАТНЫХ анона. Ну что это за хуйня блядь...
Аноним 04/06/16 Суб 10:57:37 #95 №367165 
>>367153
Скорее всего нет. Можно прогуляться по топологии, но основную часть недостающей математики лучше уже черпать из тематических курсов и книг. Никаких откровений ты больше не прочтешь, а что-то недостающее наверстаешь когда понадобится.
Группы с функаном можно, конечно, поковырять для интересу и сомообразования, но лучше лезть в численные и приближенные методы и дрочить кодинг.

И да, я не математик, математики не умеют различать полезные абстракции от бесполезного воспаления фантазий.
Аноним 04/06/16 Суб 11:42:11 #96 №367167 DELETED
>>367162
Давайте с dxdy прямо в наш тред общаться людей пригласим? :)
Аноним 04/06/16 Суб 11:48:31 #97 №367168 DELETED
>>367167
Хуже тред не станет.
МУНИН Аноним 04/06/16 Суб 12:04:08 #98 №367174 DELETED
Здравствуйте. Извольте, пока мне тут не дадут повышенные права и модерку, я с вами, животными, говорить отказываюсь. Только на dxdy. Точка.
Аноним 04/06/16 Суб 12:34:39 #99 №367178 DELETED
>>367174
Ты не мудин.
>я с вами, животными
Слишком толсто для мудина.
Аноним 04/06/16 Суб 12:52:53 #100 №367181 
Гайз, поясните ньфагу. Икосаэдр же не гомеоморфен сфере? Или гомеоморфен?
Аноним 04/06/16 Суб 12:53:42 #101 №367182 
Мне кажется, математиков в РФ можно быть только за идею. На Западе вполне можно на должности доцента или даже профа выше, чем по стране, получать.
Да и нет тут работающих математиков. Только воннаби, домоседы сраные, мнящие себя икспертами.
Аноним 04/06/16 Суб 12:54:25 #102 №367183 
>>367181
Конечно, да. Он не диффеоморфен.
Аноним 04/06/16 Суб 13:05:13 #103 №367185 
>>367183
Это потому, что икосаэдр не гладкий?
Аноним 04/06/16 Суб 13:48:35 #104 №367190 
>Он не диффеоморфен.
С какой стати?
Аноним 04/06/16 Суб 14:46:38 #105 №367195 
>>366963 (OP)
Быстра блять, какой раздел математики изучает нашу реальность напрямую? Топология?
Аноним 04/06/16 Суб 14:48:20 #106 №367196 
>>367195
Физика.
Аноним 04/06/16 Суб 14:58:24 #107 №367198 
>>367196
Не во всех учебниках по физике есть вводный курс в математику. Открываешь черные дыры и прочий майндфак-уже идут какие-то ебанутые формулы. системы и хуяк хуяк.
Аноним 04/06/16 Суб 14:59:15 #108 №367199 
>>367195
Очевидно, метаматематика.
Аноним 04/06/16 Суб 15:00:23 #109 №367200 
>>367198
Физика - это раздел математики. Предполагается, что ты достаточно компетентен, чтобы самостоятельно доказать всё, что тебе нужно.
Аноним 04/06/16 Суб 15:02:38 #110 №367201 
>>367200
>Физика - это раздел математики
На оборот же!
- Мимо Арнольд
Аноним 04/06/16 Суб 15:06:29 #111 №367202 
>>367201
Арнольд, вы умерли. Отстаньте.
Аноним 04/06/16 Суб 15:17:02 #112 №367204 
>>367185
Это ерунда.
> икосаэдр не гладкий
Гладкость - это структура, которую мы задаём, а не что-то данное свыше. Мы можем её просто перенести со сферы через гомеоморфизм, тогда всё будет диффеоморфно.
Аноним 04/06/16 Суб 16:01:41 #113 №367220 
Невозможно, пишет Беркли, представить себе мгновенную скорость, то есть скорость в данное мгновение и в данной точке, ибо понятие движения включает понятия о (конечных ненулевых) пространстве и времени.

Как же с помощью анализа получаются правильные результаты? Беркли пришёл к мысли, что это объясняется наличием в аналитических выводах взаимокомпенсации нескольких ошибок, и проиллюстрировал это на примере параболы. Как ни странно, некоторые крупные математики (например, Лагранж) согласились с ним.

Сложилась парадоксальная ситуация, когда строгость и плодотворность в математике мешали одна другой. Несмотря на использование незаконных действий с плохо определёнными понятиями, число прямых ошибок было на удивление малым — выручала интуиция. И всё же весь XVIII век математический анализ бурно развивался, не имея по существу никакого обоснования. Эффективность его была поразительна и говорила сама за себя, но смысл дифференциала по-прежнему был неясен. Особенно часто путали бесконечно малое приращение функции и его линейную часть.

В течение всего XVIII века предпринимались грандиозные усилия для исправления положения, причём в них участвовали лучшие математики столетия, однако убедительно построить фундамент анализа удалось только Коши в начале XIX века. Он строго определил базовые понятия — предел, сходимость, непрерывность, дифференциал и др., после чего актуальные бесконечно малые исчезли из науки. Некоторые оставшиеся тонкости разъяснил позднее Вейерштрасс. В настоящее время термин «бесконечно малая» математики в подавляющем большинстве случаев относят не к числам, а к функциям или последовательностям.

Как иронию судьбы можно рассматривать появление в середине XX века нестандартного анализа, который доказал, что первоначальная точка зрения — актуальные бесконечно малые — также непротиворечива и могла бы быть положена в основу анализа. С появлением нестандартного анализа стало ясно, почему математики XVIII века, выполняя незаконные с точки зрения классической теории действия, тем не менее получали верные результаты.
Аноним 04/06/16 Суб 17:10:27 #114 №367233 
Матаны, поясните ради интереса в каком разделе математики изучают low pass фильтры и их связь с moving average. В электронике все просто и интуитивно - RC, высокие частоты коротятся через кондер, пропускаются только низкие, отсюда название. Как от конденсаторов пришли к математике? Я смотрел в вики все эти формулы, экспоненциальность совершенно не интуитивна. Казалось бы какая связь между обычной RC цепочкой и ее моделью в виде exponential moving average.
Аноним 04/06/16 Суб 17:17:48 #115 №367235 
>Как иронию судьбы можно рассматривать появление в середине XX века нестандартного анализа, который доказал, что первоначальная точка зрения — актуальные бесконечно малые — также непротиворечива и могла бы быть положена в основу анализа. С появлением нестандартного анализа стало ясно, почему математики XVIII века, выполняя незаконные с точки зрения классической теории действия, тем не менее получали верные результаты.

Хуйня жи. Нестандартный анализ очень слабо связан с Лейбницевым "dx".
Аноним 04/06/16 Суб 17:21:28 #116 №367236 
>>367195
Никакой. У математики в принципе нету связи с реальностью, да и понятия реально-нереально в математике нету как такового. Реальность изучают точные науки, используя математику как один из инструментов.

>>367200
Молоток возомнил себя архитектором, лол.
Аноним 04/06/16 Суб 17:25:35 #117 №367237 
>>367236
Так а в чём пафос быть физиком? Они-то просто подгоняют модели под результаты экспериментов и ставят новые эксперименты. Никакой глубины в этом нету.
Аноним 04/06/16 Суб 18:24:32 #118 №367244 
>>367233
как выводится время разряда конденсатора через сопротивление?
Зельдович - высшая математика для начинающих.
Аноним 04/06/16 Суб 19:12:13 #119 №367249 
>>367190
Укажи этот диффеоморфизм.
Аноним 04/06/16 Суб 20:43:45 #120 №367254 
>>367236
Опять начинаешь, ебучий шакал.
>У математики в принципе нету связи с реальностью
Такое дегенеративное утверждение мог произнести только физический пропагандон.
>да и понятия реально-нереально в математике нету как такового
Ещё одна часть мифа-псевдоопределения «математики», спорить с подобными описаниями-определениями — дело сомнительное, но, тем не менее.
Есть, ещё как. Парадокс Банаха-Тарского почему называется «парадоксом»?
Математики не изучают произвольные структуры (как должны были бы делать, если бы игнорировали так называемую «реальность»), математические образы чрезвычайно наглядны и суггестивны. Все они исходят из единого источника с физикой, только точнее и яснее. Что поделать, если вкус у математиков тоньше, а воображение — богаче, разве это недостаток?
>используя математику как один из инструментов.
Как единственный нетривиальный „инструмент“. Физика — это такой монстр франкенштейна, сшитый из краденых обрывков настоящей фундаментальной науки — математики, опьянённый лоботомит, гарцующий между двумя противоположными стульями, то подставляя очко публике для сбора средств, то эксплуатируя единственный источник доброго, чистого, светлого.
Аноним 04/06/16 Суб 20:54:08 #121 №367255 
>>367254
>подставляя очко публике для сбора средств
Как и математика-кукаретика, как и вообще все науки сейчас. Аутисты могут говорить о том что НАУКА НУЖНА САМА ДЛЯ СИБЯ!! сколько угодно, но они будут сидеть без денег если не объяснят серьёзным дядям зачем это всё нужно.
Аноним 04/06/16 Суб 20:56:42 #122 №367256 
Так мы и так сидим без денег.
мимоаутист
Аноним 04/06/16 Суб 21:07:48 #123 №367258 
>>367254
Ебанутся, кем надо быть чтобы такую хуйню городить? Вас там на мехмате промывают что ли?
Аноним 04/06/16 Суб 21:13:21 #124 №367259 
> если использовать радикалы - решения уравнений x n =a, то можно явно получить решения произвольных уравнений 2-й, 3-й и 4-й степеней. Уже уравнения 5-й степени неразрешимы в радикалах.
О чём он?
Аноним 04/06/16 Суб 21:13:59 #125 №367260 
>>367259
О теореме Абеля о неразрешимости в радикалах уравнений выше четвёртой степени, видимо.
Аноним 04/06/16 Суб 21:24:02 #126 №367263 
>>367260
Не понял доказательство с википедии. Поясни для человека прошедшего только школьную программу. Он говорит что решение у уравнений ax^5+bx^4+cx^3+dx^2+ex+f=0 не может быть иррациональным. Но ведь если x=0, то решение есть при f=0. А ноль это радикал если верить определению из википедии.
Аноним 04/06/16 Суб 21:26:09 #127 №367264 
>>367263
>ax^5+bx^4+cx^3+dx^2+ex+F=0
Аноним 04/06/16 Суб 21:27:20 #128 №367265 
>>367264
Что не так? Это уравнение 5ой степени. Составил его по аналогии с квадратным.
Аноним 04/06/16 Суб 21:28:05 #129 №367266 
>>367263
Теорема утверждает, что для уравнения ax^5+bx^4+cx^3+dx^2+ex+f=0 нельзя написать общую формулу, зависящую только от a,b,c,d,e,f и содержащую только операции +-*/ и корни любой степени. В то время как для ax^2+bx+c = 0 такую формулу написать можно: x= (-b +- sqrt(b^2-4ac)) / 2a.
Аноним 04/06/16 Суб 21:29:37 #130 №367267 
>>367265
Элемент без переменной тоже должен быть нулем.
Тобишь, решение есть, но только если и переменная равна нулю и константа без переменной равна нулю. А это не распространяется на ВСЕ уравнения пятой степени.
Аноним 04/06/16 Суб 21:35:21 #131 №367268 
>>367267
Это частный случай который опровергает терему.

>>367266
Правда? А как тогда их решают? И почему именно начиная с 5ой степени?
Аноним 04/06/16 Суб 21:37:32 #132 №367269 
>>367268
Если так судить, то вообще все уравнения разрешимы, если их элементы равны нулю.
Но какова мощность у такого множества уравнений, по сравнению с остальными...
Аноним 04/06/16 Суб 21:39:47 #133 №367270 
>>367268
>Правда? А как тогда их решают?
Численно, когда нужно проводить какие-то рассуждения просто говорят "пусть x0 - какой-то из корней такого уравнения" и проводят рассуждения с ним (так делают и для квадратных уравнений, btw). На самом деле "в реальном мире" очень редко когда что либо можно аналитически решить, поэтому это не очень удивительно.
>И почему именно начиная с 5ой степени?
Фундаментальная причина этого: это то, что S_4 (группа перестановок из 4х элементов) ещё содержит нетривиальные нормальные подгруппы, а S_5 уже нет (под тривиальными я понимаю и знакопостоянную A_n которая есть у любой S_n).
Аноним 04/06/16 Суб 21:40:16 #134 №367271 
>>367269
Теорема не доказана если есть частный случай её опровергающий.
Аноним 04/06/16 Суб 21:41:28 #135 №367273 
>>367271
Теорема не утверждает, что любое уравнение выше пятой степени неразрешимо в радикалах, она утверждает, что нельзя выписать одну общую формулу для всех уравнений.
Аноним 04/06/16 Суб 21:46:41 #136 №367274 
>>367273
Понял это. Он тут >>367267 сказал, что это не считается т.к. частный случай.
Хотя ещё 1 вопрос. Это говорит теорема Абеля или то предложение которое я запостил? Просто в том предложении не говорится про отсутствие формулы, а говорится именно то, что уравнение выше пятой степени неразрешимо в радикалах. Это ошибка?

>>367270
Ничего не понял, но всё равно спасибо.
Аноним 04/06/16 Суб 21:48:45 #137 №367275 
>>367274
Нету ОБЩЕГо решения.
Для тупых, с каждым уравнением пятой степени надо ебаться в сракотан, каждый раз, с каждым уравнением, по разному, чтобы найти решение.

Для всех уравнений степенью ниже есть формулы в которые просто надо поставить цифры и посчитать столбиком.
Аноним 04/06/16 Суб 21:50:05 #138 №367276 
>>367274
https://ru.wikipedia.org/wiki/%D0%A0%D0%B0%D0%B4%D0%B8%D0%BA%D0%B0%D0%BB
Аноним 04/06/16 Суб 21:50:25 #139 №367277 
>>367274
Под неразрешимостью в радикалах и понимают отсутствие формулы. Правда нужно было добавить "уравнение выше четёртой степени в общем виде не разрешимо в радикалах", чтобы уже совсем придирок не было.
>Ничего не понял, но всё равно спасибо.
Есть хорошая книга Алексеева "Теорема Абеля в задачах и решениях", как раз для уровня "постшкольника". Если тебя это правда заинетересовало - можешь почитать её (там не только теорема Абеля, но и много хорошей и глубокой математики вокруг неё).
Аноним 04/06/16 Суб 21:51:30 #140 №367278 
>>367277
Понял. Спасибо.
Аноним 04/06/16 Суб 21:53:09 #141 №367279 
>>367258
Промыли тут тебя, и неплохо промыли.
Аноним 04/06/16 Суб 22:11:36 #142 №367283 
А есть ведь разрешимость не в радикалах, а в каких-то других функциях. Почему бы не использовать их?
Аноним 04/06/16 Суб 22:14:29 #143 №367284 
>>367283
Разумеется мы можем сделать многозначную функцию "F(a,b,c,d,e,f)=все решения уравнения ax^5+bx^4+cx^3+dx^2+ex+f=0" и исследовать её. И получать о ней даже какие-то содержательные аналитические утверждения: приближения, разложения в ряды, неравенства, быстрые алгоритмы, вычисляющее её, и так далее. Так и делают, собственно.
Аноним 04/06/16 Суб 22:19:46 #144 №367286 
>>367283
А, я чуть по-другому твой вопрос воспринял. Интуиция нам подсказывает, что если формулы в радикалах не существует, то и формулы, использующей всякие экспоненты, логарифмы, синусы/косинусы и прочие стандартные школьные функции тем более не существует, но строгих теорем на этот счёт нету.
Аноним 04/06/16 Суб 22:40:53 #145 №367287 
>>367279
На дхду съеби. Там ты будешь чувствовать себя в своей стихии, как говно в унитазе.
Аноним 04/06/16 Суб 22:43:32 #146 №367288 
>>367284
>>367286
Имеется ввиду набор функций, которые явно заданы, такой что существует формула для любого полиномиального уравнения. Насколько я нагуглил, для гипергеометрических и тета функций потенциально такие формулы существуют.
Аноним 04/06/16 Суб 22:45:27 #147 №367289 
>>367288
Чето как на школьника ты слишком много знаешь.
Тонкий что ли?
Аноним 04/06/16 Суб 22:45:45 #148 №367290 
>>367289
Это другой.
Аноним 04/06/16 Суб 22:45:56 #149 №367291 
>>367288
Тут вся проблема в том, что считать под "явно заданы". Вот (арифметический) квадратный корень (вещественного) неотрицательного числа определяется как положительное решение уравнения x^2 = a. Насколько это можно считать "явным заданием"?
>Насколько я нагуглил, для гипергеометрических и тета функций потенциально такие формулы существуют.
Не знаю ничего об этом, вполне возможно.
Аноним 04/06/16 Суб 22:57:43 #150 №367297 
>>367287
А вот и оскорбления пошли, ещё dxdy приплёл. Там что, кто-то озвучивал схожие взгляды?
Аноним 04/06/16 Суб 23:40:59 #151 №367302 
>>367254
>математические образы чрезвычайно наглядны и суггестивны
Сколько можно уже это обоссывать? Математика зиждется на теории множеств и дальнейшей метаматематике, которая представляет собой скопление совершенно ненаглядных и трудных для восприятия образов. Притом к реальности и интуитивности эти области совершенно не тяготеют, им негде черпать эту реальность.
Аноним 04/06/16 Суб 23:43:03 #152 №367303 
>>367297
dxdy говно, там пидоры. дискас.
Аноним 04/06/16 Суб 23:46:31 #153 №367305 
>>367291
Неформально - функции, которые приходят из других теорий, в контексте которых есть рецепты их вычисления.
Если так хочется формально - в разложении в ряд явно заданы коэффициенты.
Аноним 04/06/16 Суб 23:48:14 #154 №367306 
>>367302
>Математика зиждется на теории множеств
Абсолютно неправда, теория множеств нужна только как общий язык для обоснования основ математики, и без неё прекрасно обходились вплоть до конца ХХ века. Если завтра логики выяснят, что теория множеств иррелевантна по каким-то их логическим причинам (например, окажется противоречивой), то 99.99% математических результатов всё равно сохранятся и им будет абсолютно похуй. Знаю мощного аналитика, который не знает, что такое кардинал и кроме алеф-нуль и континуума мощностей не знает. И жив как-то.
Аноним 04/06/16 Суб 23:52:01 #155 №367307 
>>367305
Ну так для функции "F : R^5 -> R^5" которая равна всем корням уравнения общего уравнения пятой степени тоже можно вычислить коэффициенты разложения в ряд тейлора по теореме о неявной функции. По крайней мере в тех точках, где все корни различны.
Аноним 04/06/16 Суб 23:55:56 #156 №367308 
>>367306
>Абсолютно неправда, теория множеств нужна только как общий язык для обоснования основ математики, и без неё прекрасно обходились вплоть до конца ХХ века.
Расскажи мне поподробнее как без нее обходились и кто все-таки припилил к математику эту нелепую канторову выдумку?

>Если завтра логики выяснят, что теория множеств иррелевантна по каким-то их логическим причинам (например, окажется противоречивой), то 99.99% математических результатов всё равно сохранятся и им будет абсолютно похуй. Знаю мощного аналитика, который не знает, что такое кардинал и кроме алеф-нуль и континуума мощностей не знает. И жив как-то.
Ну какой-нибудь отшельник тоже может революции не заметил, но это же ничего не говорит о революции.
Аноним 05/06/16 Вск 00:05:54 #157 №367309 
>>367305
>Расскажи мне поподробнее как без нее обходились
А что нужно от теории множеств рядовому математику? Конструкция функции, отношения эквивалентности и фактора по нему, декартового произедения множеств и всё. Всё это можно объяснить нетупому школьнику 7-8 класса абсолютно не вдаваясь ни в какие аксиоматические детали.

>Ну какой-нибудь отшельник тоже может революции не заметил, но это же ничего не говорит о революции.
Это не единичный случай, а типичный; может этот слегка гипертрофирован тем, что хорошие математики всё же знает поглубже теоретико-множественный фреймворк, но абсолютно адекватен тому факту, что математику можно его не знать. Факт в том, что в теории множеств, как и в понятии о множестве нету ничего фундаментального. Вместо теории множеств могли быть типы и конструкторы типов (что сейчас продвигается некоторыми чуваками), могли быть точки и стрелки (что сейчас продвигаетсся категорщиками), могли быть фундированные деревья, могло быть что угодно - математику можно строить на любом достаточно гибком лингвистическом фреймворке, результаты о том, что существует ровно пять платоновых тел и ровно 17 wallpaper groups (из которых и состоит математика) не потеряют ни капли своей значимости, глубины и даже "природности" от этого.
Аноним 05/06/16 Вск 00:06:56 #158 №367310 
>>367307
Как вычислить?
Аноним 05/06/16 Вск 00:15:14 #159 №367311 
Я прошу прощения. На отрезке [a, b] у точек a и b же нет ненулевых окрестностей в [a, b]?
Аноним 05/06/16 Вск 00:22:51 #160 №367312 
>>367309
опять же, ты твоя аргументация вообще ничего не проясняет. Ну и что, что она не нужна какому-то определенному математику? Она это этого перестала быть частью математики, перестала "отражать реальность и черпать из нее"?
Аноним 05/06/16 Вск 00:30:40 #161 №367313 
>>367311
Есть, например [a,c) и (c,b], где a<c<b. Окрестности всегда есть, всё пространство — окрестность каждой своей точки.
Аноним 05/06/16 Вск 00:34:06 #162 №367314 
>>367313
Вот это поворот. А я почему-то думал, что окрестности должны быть симметричны во всех направлениях и представлять на плоскости внутренность окружности, а на прямой - равные интервалы в обе стороны от точки.
Аноним 05/06/16 Вск 00:49:22 #163 №367315 
>>367310
Покажу только для случая n=2
Рассмотрим функцию f:R^2->R^2 которая отображает (b,c) -> (x1,x2) коэффициенты уравнения x^2+bx+c в его корни.
У нас есть система уравнений вида
F = 0
F(b,c,x1,x2): R^4 -> R^2

F1(b,c,x1,x2) = c - x1x2
F2(b,c,x1,x2) = b + x1 + x2

теорема о неявной функции утверждает что в окрестности хороших точках существуют единственные гладкие функции x1(b,c) x2(b,c) которые решают это уравнение F(b,c,x1(b,c),x2(b,c))=0
при этом дифференциал функции f' = - F'_ab / |F'_x1x2|, иначе говоря, в координатах это будет выглядеть так:
(dx1/da, dx1/db)
(dx2/da, dx2/db) =
-
(dF1/dx1 dF1/dx2)^-1
(dF2/dx1 dF2/dx2)
x
(dF1/da dF1/db)
(dF2/da dF2/db)

вот и первый член ряда тейлора.
>>367312
Тогда я не очень понял твоей мысли; я думал твой тезис примерно следующий: "математика часть теории множеств, теория множеств абстрактна и мало связей имеет с реальным миром, следовательно, математика абстрактна и не имеет связи с реальным миром". Так вот, у теории множеств и математики (а именно: геометрии, алгебры и анализа) связей примерно столько же, сколько у лингвистики и литературы. Если завтра лингвисты введут новую систему падежов, то на произведениях Кафки это никак не отразится.
Аноним 05/06/16 Вск 00:49:58 #164 №367316 DELETED
Обсуждают какое-то говно, как в лужу пердят. Какой-то сраный философия-тред, а не математика. На вопросы по делу ИТТ 1.5 тупорылых воннаби, выучивших 1.5 термина из теории катов, ответить вообще не в состоянии. Их недознаний хватает только на тупые философские срачи. Говно, а не математика-тред.
Аноним 05/06/16 Вск 00:51:37 #165 №367317 DELETED
>>367316
А по-моему охуенный. А по поводу философии вообще претензии непонятны, уж кому-кому, а математикам (в отличии от химиков, медиков, нефундаментальных физиков) свою деятельность осмыслять уж точно нужно.
Аноним 05/06/16 Вск 00:52:59 #166 №367318 DELETED
>>367317
Приведи пример хоть одного интересного вопроса, который тут обсуждён, кроме философского пердежа и обзываний друг друг картохой.
Аноним 05/06/16 Вск 00:53:24 #167 №367319 DELETED
>>367318
Приведи пример того, что ты считаешь хорошим вопросом.
Аноним 05/06/16 Вск 00:56:48 #168 №367320 DELETED
>>367319
Ясно. Вопросом на вопрос.
Аноним 05/06/16 Вск 00:57:49 #169 №367321 DELETED
>>367320
А что в этом плохого? Если я не до конца понял вопрос, то как я на него могу ответить? В чём зашкварность задавать уточняющие вопросы?
Аноним 05/06/16 Вск 01:01:15 #170 №367322 DELETED
>>367321
Приведи пример вопроса, который по-твоему интересен.
Аноним 05/06/16 Вск 01:07:18 #171 №367325 DELETED
>>367322
Роль теории множеств в современной математике, разложение корней уравнения степени n в ряд тейлора, может для кого-то это было сверхтривиально, но мне было интересно почитать.
Аноним 05/06/16 Вск 01:07:37 #172 №367326 
>>367314
На отрезке [a,b] шар радиуса r, меньшего b-a, с центром в точке a выглядит так: [a,a+r). Шар, определённый с помощью расстояния — это множество точек, расстояние которых до его центра меньше радиуса. Окрестность точки должна содержать шар с центром в этой точке, если топология определена расстоянием, конечно.
Аноним 05/06/16 Вск 01:09:17 #173 №367327 DELETED
>>367325
Пиздец.
Аноним 05/06/16 Вск 01:10:01 #174 №367328 DELETED
>>367327
Ты просто зануда ёбанный по жизни, которому лишь бы поныть, ненавижу таких.
Аноним 05/06/16 Вск 01:12:28 #175 №367329 DELETED
>ёбанный
Ну ладно, и то хорошо, что этот просто школота тупорылая.
Аноним 05/06/16 Вск 01:14:07 #176 №367330 DELETED
>>367329
Пошёл ннахуй.
Аноним 05/06/16 Вск 01:15:33 #177 №367331 DELETED
Хорошо, что такой мусор никогда не пробъётся в науку, тем более западную. Я спокоен.
Аноним 05/06/16 Вск 01:15:54 #178 №367332 DELETED
>>367331
Пошёл нахуй.
Аноним 05/06/16 Вск 01:17:28 #179 №367333 DELETED
Как школьника-то порвало. Сидит, аж трясётся весь. Тупая жертва ЕГЭ. Корни в ряд Тейлора раскладывает, охлол блядь.
Аноним 05/06/16 Вск 01:20:26 #180 №367334 DELETED
>>367333
Пошёл нахуй.
Аноним 05/06/16 Вск 01:21:59 #181 №367335 DELETED
Ух, ты! У нас новый срач! Давно такого не было!
Аноним 05/06/16 Вск 01:22:31 #182 №367336 DELETED
20 раз напиши, так не достаточно.
Аноним 05/06/16 Вск 01:23:25 #183 №367337 DELETED
>>367336
Пошёл нахуй.
Аноним 05/06/16 Вск 01:29:39 #184 №367339 
>>367315
ты все время пытаешься у меня найти аргумент, что теория множеств определяет или даже является (что вообще абсурд) основой математической науки. Я же утверждаю, что она является частью этой науки, причем немаловажной, но тем не менее не черпает свои знания из реальности. Мы же не можем выкинуть просто так теорию множеств, или найти ей реальную замену. Значит и математика не может быть наукой о реальности, раз содержит такую часть.
Аноним 05/06/16 Вск 01:31:57 #185 №367340 DELETED
Ну ты прям у мамы Сократ, ёпт.
Аноним 05/06/16 Вск 01:32:22 #186 №367341 
>>367326
Но это же половина шара. А как же точки с координатой c<a<b? Они должны же лежать в шаре, чтобы он получился шаром. Как же я туп(
Аноним 05/06/16 Вск 01:33:13 #187 №367342 
>>367339
> Значит и математика не может быть наукой о реальности, раз содержит такую часть.
Очень странное суждение, как по мне. В фундаментальной физике есть toy models с n пространственными и m временными измерениями, которые тоже к реальности отношения не имеют, разве из этого следует, что физика не может быть наукой о реальности, если содержит такую часть?
Аноним 05/06/16 Вск 01:43:38 #188 №367344 
>>367339
>не черпает свои знания из реальности
О какой реальности идёт речь? Понятия теории множеств вполне наглядны — конечное множество букв алфавита, счётное множество связано с образом натурального ряда, континуум — вещественная линия, дальше идёт пространство, точками которого являются функции на прямой. Декартово произведение, множество подмножеств — всё это понятия, которые можно легко иллюстрировать.
Аноним 05/06/16 Вск 01:52:28 #189 №367345 
>>367342
Я же не зря отметил неотьемлемость ТМ, что потеряет физика без этих моделей? Математике же все равно нужен метааппарат (хоть какой-то), иначе непонятно насколько ее суждения будут истинны и проверяемы.

>>367344
В чем состоит отличие множеств:
{{},{},{}} {{{}},{},{}} {{{},{}},{}} {}
и как это отражается в реальности?
А как иллюстрировать понятие мощность бесконечного множества какой-либо реальной аналогией?
Ну и про наглядность Банаха-Тарского тоже очень интересно. Все-таки это не парадокс, а верное суждение.
Аноним 05/06/16 Вск 02:09:07 #190 №367347 
>>367345
>Математике же все равно нужен метааппарат (хоть какой-то), иначе непонятно насколько ее суждения будут истинны и проверяемы.
Ты опять же воспринимаешь теорию множеств как "аппарат для математики", мне кажется это несколько некорректно. Мне кажется до того, как Бурбаки расфорсили ТМ, тоже все прекрасно без неё обходились.


>В чем состоит отличие множеств:
>{{},{},{}} {{{}},{},{}} {{{},{}},{}} {}
>и как это отражается в реальности?
Это странный вопрос, отражается тем, что коробка, в которой есть пустая коробка это не то же самое, что коробка, в которой коробка, в которой пустая коробка.
>А как иллюстрировать понятие мощность бесконечного множества какой-либо реальной аналогией?
Отрезок и натуральный ряд, вполне реальные аналогии, как по мне.
>Ну и про наглядность Банаха-Тарского тоже очень интересно. Все-таки это не парадокс, а верное суждение.
Мне кажется, что парадокс Банаха-Тарского просто иллюстрирует, что при интерпретации мат. моделей (и, соответственно, мат. теорем) на реальность нужно быть аккуратнее. Это и так было понятно: например уравнения "показывают" что игла может стоять вертикально на столе, тогда нужно учитывать ещё и такое свойство как "устойчивость", но и не все устойчивые решения могут иметь физический смысл, тогда нужно учитывать что-то ещё, и так далее.
Аноним 05/06/16 Вск 03:13:20 #191 №367349 
>>367254
>Кудах-бабах!!
Банах-Тарский как пример связи с реальностью, лол. Уж лучше бы молчал.

>>367255
Математики-кукаретики так не делают. Я уже показывал в каком-то старом треде на конкретных примерах (ERC) как математикам-кукаретикам приходится подставлять очко физикам, химикам и биологам, а те уже работают с публикой. Чистого математика публика даже ебать не станет, а другие науки делают это исключительно из жалости.

>>367258
Это не специально, это побочка от перегруженности мозга математикой.

>>367342
toy models это немножко не то что ты думаешь. Это продукт ленивости физиков и от строгой математики он сидит еще дальше чем простые модели.

>>367347
> коробка, коробка
Только вот коробок в тех примерах нету. Это тебе как математику наличие коробки эквивалетно отсутствию, ты просто не можешь различить эти два случая. Для тебя "коробка" - абстрактная хуета, которую можно добвить или убрать в зависимости от того какой способ удобней для понимания конструкции.

Аноним 05/06/16 Вск 03:17:15 #192 №367350 
>>367349
>Только вот коробок в тех примерах нету. Это тебе как математику наличие коробки эквивалетно отсутствию, ты просто не можешь различить эти два случая. Для тебя "коробка" - абстрактная хуета, которую можно добвить или убрать в зависимости от того какой способ удобней для понимания конструкции.

Не понял. Вложенные относятся к абстракции конечного множества так же, как и линия нарисованная карандошом к абстракции прямой в геометрии - то есть это просто очень хорошая аналогия неоторого математического конструкта.
Аноним 05/06/16 Вск 03:19:58 #193 №367351 
>>367350
Вложенные коробки*
Аноним 05/06/16 Вск 03:38:48 #194 №367353 DELETED
>>367335
один из участников срача кажется туповат - не знает как отвечать на посты
Аноним 05/06/16 Вск 03:43:12 #195 №367354 DELETED
>>367353
Пошёл нахуй.
Аноним 05/06/16 Вск 05:23:02 #196 №367355 
https://www.youtube.com/watch?v=IU6zuPi2Epk
В видео типичный физик.
Аноним 05/06/16 Вск 05:24:29 #197 №367356 
>>367283
Их используют. Просто никто уже давно не занимается решением уравнений степени n над полем вещественных чисел - давно всё решили.
Аноним 05/06/16 Вск 05:31:59 #198 №367357 
>>367339
>но тем не менее не черпает свои знания из реальности
Канон теории множеств прямо зависит от реальности. Потенциальных аксиом теории множеств так много, что они даже не образуют множество; но избраны из них только девять. Как раз из-за реальности.
Аноним 05/06/16 Вск 05:47:47 #199 №367358 
>>367357
Образуют.
Аноним 05/06/16 Вск 07:22:31 #200 №367360 
>>367358
Нет, не образуют. Их можно занумеровать ординалами.
Аноним 05/06/16 Вск 08:00:32 #201 №367363 
>>367360
Что ты ординалами нумеровать собрался? Подмножества множества всех строк над алфовитом логики первого порядка?
Аноним 05/06/16 Вск 08:08:34 #202 №367365 
>>367363
Даже ZFC не является чистой теорией первого порядка, в ней есть схемы аксиом. Почему бы не рассмотреть схемы схем, схемы схем схем и так далее? Технически это означает построение w-логики для каждого ординала w.
Аноним 05/06/16 Вск 08:18:18 #203 №367366 
>>367365
Я может что-то не понимаю, но всегда думал, что схема аксиом - это просто вычислимое подмножество аксиом, нет?
Аноним 05/06/16 Вск 08:32:43 #204 №367368 
>>367366
Схема аксиом языка X - это метаматематический текст, который считается достаточно понятным, чтобы породить бесконечное множество формул языка X (которые называются соответствующими схеме). Эти формулы затем объявляются аксиомами.
Аноним 05/06/16 Вск 08:37:25 #205 №367371 
>>367368
Всё-таки если определять чуть более строго, то это множество формул порождаемое некоторым алгоритмом, то есть - вычислимое. В любом случае, что в твоём варианте, что в моём, это никак нельзя назвать чем-то, что принадлежит логике второго порядка.
Аноним 05/06/16 Вск 08:53:02 #206 №367374 
>>367371
Определить "алгоритм" очень сложно, и при попытках определить его всё равно всплывут слова "понятный метаматематический текст". Ну и зачем тогда использовать понятие алгоритма для определения схемы аксиом?
Аноним 05/06/16 Вск 10:17:24 #207 №367382 
>>367316
Просто тред математики отобразился сам в себя через функцию треда математики для начинающих
Аноним 05/06/16 Вск 10:40:36 #208 №367383 
Какой морфизм должен быть у категории, состоящей из бесконечных множеств, чтобы она была полной подкатегорией категории set.
Единственное что лезет в голову морфизм hom(a,b)= b если |a|<|b|
Аноним 05/06/16 Вск 10:58:07 #209 №367385 
Жду клоуна с диффеоморфизмом икосаэдра в шар.
Аноним 05/06/16 Вск 11:22:08 #210 №367387 
>>367383
Те же морфизмы, что и раньше — отображения из A в B.
>>367385
Икосаэдр вообще не гладкое многообразие, поэтому в принципе непонятно, что понимать под «диффеоморфизмом».
Аноним 05/06/16 Вск 11:25:37 #211 №367388 
>>367349
>Банах-Тарский как пример связи с реальностью, лол.
Употребление слова „парадокс“ в названии математической теоремы как пример связи с реальностью мышления математиков.
Аноним 05/06/16 Вск 11:50:08 #212 №367394 
>>367387
>>367190
Аноним 05/06/16 Вск 11:54:24 #213 №367395 
>>367394
И?
Аноним 05/06/16 Вск 12:05:30 #214 №367402 
Кажется у нас появился Set-Петух
Аноним 05/06/16 Вск 12:24:53 #215 №367405 
>>367395
И он сказал, что, мол, диффеоморфизм есть.
Аноним 05/06/16 Вск 13:01:40 #216 №367408 
>>367405
Где?
Аноним 05/06/16 Вск 13:25:41 #217 №367410 
>>367408
Заебал, дебил.
Аноним 05/06/16 Вск 13:32:31 #218 №367411 
>>367402
Подготовьте пики.
Аноним 05/06/16 Вск 13:58:22 #219 №367419 
>>367410
Зеркальное приписывание своих промахов другим — не самая лучшая стратегия.
Аноним 05/06/16 Вск 14:35:47 #220 №367429 
Оторвитесь от определения N, помогите решить домашку.
Топология, нужно построить атлас на грассманиане (2, 5)
Прочитал про построение карт вот тут
http://people.math.umass.edu/~tevelev/5-20.pdf
раскурил, обьяснил их преподу по этому образцу, все норм.
Теперь нужно написать функции склейки между картами, и тут я как-то плохо понимаю, что делать. Я же, вообще говоря, не знаю вроде бы как у меня выглядят матрицы A_{ij}, чтобы просто написать что-то вроде A_{i'j'}^{-1} A_{ij} A
Аноним 05/06/16 Вск 14:55:23 #221 №367432 
>>366963 (OP)
как найти второй диф. функции
f(a(x,y),b(x,y))
Аноним 05/06/16 Вск 15:04:27 #222 №367434 
>>367341
Гайз! Не оставляйте, плз. У меня сдвиг парадигмы намечается.
Аноним 05/06/16 Вск 16:21:59 #223 №367438 
>>367311
Что такое ненулевая/нулевая окрестность? Ответ зависит от твоего определения окрестности.

Самое хорошее определение: окрестностью точки называется любое открытое множество, содержащее точку. Любая точка имеет хотя бы одну окрестность - все пространство (см. определение топологического пространства).

Если не изучал топологические пространства, то в метрическом пространстве окрестность точки x - это открытый шар с центром в точке x.

Если не изучал метрические пространства, то eps-окрестностью точки x из R называется множество
{ y из R | |x - y| < eps }.
Аноним 05/06/16 Вск 16:30:14 #224 №367440 
>>367434
>>367438
То есть, в смысле определения 1 любая точка имеет окрестность.

В смысле определения 3 твой вопрос некорректен. Потому что в этом смысле не бывает окрестностей в чем-то, все окрестности только в R.

В определении 2 легко запутаться, если ты не понимаешь, что такое шар в метрическом пространстве, но на самом деле все легко.

Короче, если путаешься, разбирайся с определением 1, оно самое хорошее и общее.
Аноним 05/06/16 Вск 17:08:36 #225 №367446 
>>367311
Смотря какая топология. Если метрическая, то эти точки не имеют окрестности, кроме тривиальных.
Аноним 05/06/16 Вск 17:21:15 #226 №367448 
>>367195
Знания РУСов, коренного населения земли
https://www.youtube.com/watch?v=MWepwyMidqw
Аноним 05/06/16 Вск 17:33:53 #227 №367449 
>>367438
>>367440
>>367446
Ну вот есть тот же отрезок [a, b] прямой действительных чисел. Он замкнут в R. У него есть минимальное значение a, и максимальное - b. Есть ли у точек a и b ненулевая окрестность, содержащаяся в [a, b]? Собственно это вся задача, через которую я пытаюсь прийти к осознанию понятия локальной компактности и локальной связности. Это какая топология?
Аноним 05/06/16 Вск 17:38:19 #228 №367451 
>>367448
Сука, вы послушайте что они несут.
Аноним 05/06/16 Вск 17:42:46 #229 №367452 
>>367451
>Ещё раз повторяю что школьная программа о трёх мерном мире не верна, мир многомерный! Трёх мерное пространство есть в трёх мерном мире. В нашем мире минимум 5 измерительных качеств пространства. Зрение-Слух-Вкус-Осязание- Обаняние и ими мы соизмеряем всю жизнь. А вам втюривают что наш мир это только длинна ширина и высота.
Аноним 05/06/16 Вск 17:43:18 #230 №367454 
>>367385
> Жду клоуна с диффеоморфизмом икосаэдра в шар
>>367387
>Икосаэдр вообще не гладкое многообразие
Дети-детишки, новый открытия в области дифференциальной геометрии итт.
Аноним 05/06/16 Вск 17:44:25 #231 №367455 
>>367449
Ты заебал уже со своим птушным определением. Что такое ненулевая окрестность? Это во-первых. Во-вторых, ты сам должен знать какая топология, раз спрашиваешь. Если не знаешь, то вопросы об открытости/замкнутости не имеют смысла.
Аноним 05/06/16 Вск 17:48:13 #232 №367456 
>>367451
Но они правы!
https://www.youtube.com/watch?v=bjVD7c_AHz8
Аноним 05/06/16 Вск 17:48:30 #233 №367457 
>>367454
А что, икосаэдр это гладкое многообразие?
Аноним 05/06/16 Вск 17:49:36 #234 №367458 
>>367454
Ну так где там диффеоорфизм-то обещанный, уёбок?
Аноним 05/06/16 Вск 18:27:39 #235 №367462 
>>367452
>>367456
Взрослые, казалось бы, люди. шар у них, блеадь, одним измерение описывается.
Аноним 05/06/16 Вск 18:51:39 #236 №367465 
>>367374
Да нет, очень просто. Алгоритм - это частично рекурсивная функция.
Аноним 05/06/16 Вск 19:03:09 #237 №367467 
>>367465
Но ведь функция определяется как некоторое множество ZFC.
Аноним 05/06/16 Вск 19:04:28 #238 №367469 
>>367467
Да. И?
Аноним 05/06/16 Вск 19:17:17 #239 №367471 
>>367347
>Бурбаки расфорсили ТМ
Когда ТМ форсилась в математике Бурбаки еще не родились, а точкой начала ее власти можно считать первый математический конгресс. Да и это очевидно, если почитать математические труды, например, по анализу, середины 19 века и начала 20ого.

>Это странный вопрос, отражается тем, что коробка, в которой есть пустая коробка это не то же самое, что коробка, в которой коробка, в которой пустая коробка.
ну а теперь вопрос, содержится ли пустая коробка в которой пустая коробка в множестве присутствующих в этом треде? А если предположить, что это коробка, где содержится Гитлер, она получается тоже среди нас (ведь коробка пуста)?

>Отрезок и натуральный ряд, вполне реальные аналогии, как по мне.
И так, у нас есть натуральный ряд, натуральный ряд откуда мы выкинули несколько чисел, и множество всех подмножеств натурального ряда, как же нам интуитивно их расставить?

>Мне кажется, что парадокс Банаха-Тарского просто иллюстрирует, что при интерпретации мат. моделей (и, соответственно, мат. теорем) на реальность нужно быть аккуратнее.
Вполне. Однако, он затрагивает собой неизмеримые множества, аксиому выбора, свойства натурального ряда... Что и приводит к странному результату. Но где же в этом всем реальность? Откуда же она берется во всех этих преобразованиях?
Аноним 05/06/16 Вск 19:25:58 #240 №367472 
>>367469
Чтобы ввести ZFC, нужно располагать понятием схемы аксиом. Поэтому определение схемы аксиом не должно зависеть от понятия алгоритма.
Аноним 05/06/16 Вск 19:26:46 #241 №367473 
>>367457
Конечно.
>>367458
В смысле обещанный? Я тебе ничего не обещал явно построить. А ты явно гомеоморфизм построить сможешь тогда, раз утверждаешь что они гомеоморфны? Давай гомеоморфизм - получишь диффеоморфизм.
Аноним 05/06/16 Вск 19:32:09 #242 №367475 
>>367472
Лол, а к определению обычной аксиомы, строки, символа, правила вывода, терма, wwf почему подобный критицизм не относится? Когда мы строим логику, мы предполагаем, что работаем в достаточно мощной метатеории. В частности, достаточно мощной для того, чтобы ввести понятие алгоритма.
Аноним 05/06/16 Вск 19:42:24 #243 №367476 
>>367475
Нет, не предполагаем. В метаязыке, с помощью которого ZFC вводится впервые, есть только понятия "символ", "строка", "текст" (конечная последовательность строк), и с ними можно делать стандартные операции - проверять наличие данного символа в данной строке, сравнивать две данные строки, проверять наличие данной строки в данном тексте, заменять в данном тексте один символ на другой символ, совершать конкатенацию и т.п. Модели ZFC в самой ZFC - отдельный разговор.
Аноним 05/06/16 Вск 19:45:27 #244 №367477 
>>367476
Это какой-то сверхнаивный взгляд. Даже чтобы определить, что такое wwf в логике первого порядка нужно уже построить кое-какой парсер, тобишь иметь по силе нечто эквивалентное алгоритму.
Почитай на досуге http://math.stackexchange.com/questions/201703/an-apparently-vicious-circle-in-logic
Аноним 05/06/16 Вск 19:47:45 #245 №367478 
>>367477
Тем не менее, именно такой подход успешно реализован в книге Бурбаки. Понятие "символ" они, кстати, определять отказались, так как предполагали-де, что их книгу не будут читать марсиане.
Аноним 05/06/16 Вск 19:50:38 #246 №367479 
>>367478
Это уже какая-то апилляция к авторитетам.

Скажу только, что книжка Бурбаки является безусловным авторитетом в вопросах того, сколько отображений существует из пустого множества в пустое. Но это не книжка по мат. логике.
Аноним 05/06/16 Вск 19:50:55 #247 №367480 
>>367479
апелляция*
Аноним 05/06/16 Вск 19:52:56 #248 №367481 
>>367479
Поначитаются Вавилова, понимаешь.
Да, это не книжка по матлогике. Это книжка по математике, причём хорошая книжка. Я, кстати, склонен считать, что логика и математика - это вообще две разные науки, которые не имеют абсолютно ничего общего. И то, что ты не обнаружил в книге Бурбаки матлогики, служит аргументом в пользу моего мнения.
Аноним 05/06/16 Вск 19:53:14 #249 №367482 
>>367473
Тогда просто докажи, что он существует.
Аноним 05/06/16 Вск 19:58:49 #250 №367483 
>>367481
Вавилов няша :3
Твоё мнение расходится с мнением научного сообщества. Мне вполне очевидно, что математическая логика в отрыве от математики никому не интересна (что очевидно из названия).

Да мне всё равно, в принципе, понял ты или нет, в любом учебнике по логике (Cori Lascar; Манин; Мендельсон) написано ровно то же, что сказал я, и что написано на stackexchange. Если хочешь продолжать учить логику по Бурбаки и путать схемы аксиом и аксиомы в языке второго порядка - то пожалуйста.
Аноним 05/06/16 Вск 20:08:23 #251 №367485 
>>367483
Есть довольно много логиков (например, Непейвода), которые занимаются логикой, открывают всякие там логические теоремы, делают учёные и хитрые вещи, но ничего не понимают в математике. Есть, аналогично, довольно много математиков, которые знать не знают, чем модальная логика отличается от темпоральной. Этого бы не было, если бы две науки, - математика и логика, - являлись одной и той же наукой.

Логика в математике не используется никак. На первый взгляд это заявление кажется неверным, да. Но как только ты окинешь взглядом всё то, что логики создали за последние сто лет, ты с ним согласишься, я думаю.
Аноним 05/06/16 Вск 20:23:50 #252 №367487 
>>367485
>Есть довольно много логиков (например, Непейвода), которые занимаются логикой, открывают всякие там логические теоремы, делают учёные и хитрые вещи, но ничего не понимают в математике. Есть, аналогично, довольно много математиков, которые знать не знают, чем модальная логика отличается от темпоральной. Этого бы не было, если бы две науки, - математика и логика, - являлись одной и той же наукой
Это вполне нормально, что специалисты в одной области не знают некоторых специальных вещей из другой области. Не вижу, где утверждение сломается если заменить логиков на КК-теоретиков, а модальную логику на функтор Каспарова.

>Логика в математике не используется никак. На первый взгляд это заявление кажется неверным, да. Но как только ты окинешь взглядом всё то, что логики создали за последние сто лет, ты с ним согласишься, я думаю.
Я не очень понимаю, что ты понимаешь под "математикой". В любом случае, помимо логики есть достаточно мощные попарно не использующие друг друга разделы. Btw, вот такой вот пост нагуглил http://mathoverflow.net/questions/7018/model-theoretic-applications-to-algebra-and-number-theoryiwasawa-theory
Аноним 05/06/16 Вск 20:29:21 #253 №367489 
>>367487
В таком случае вообще всю науку логиков придётся записать в "некоторые специальные вещи", потому что она вся является неведомой для математиков.
Аноним 05/06/16 Вск 20:29:55 #254 №367490 
>>367489
Ссылку открывал, не?
Аноним 05/06/16 Вск 20:55:36 #255 №367492 
Почему SU(2) компактная группа?
Аноним 05/06/16 Вск 20:59:10 #256 №367494 
>>367482
Диффеоморфизм - это гомеоморфизм, который переводит гладкие функции в гладкие. Задаём гладкую структуру (т.е. атлас) на икосаэдре так, что гладкие функции на нём - это гладкие функции на сфере как на гомеоморфном образе. Тогда этот гомеоморфизм - диффеоморфизм по построению.
Аноним 05/06/16 Вск 21:10:55 #257 №367495 
Выскажусь по вопросу "является ли логика математикой"?

Математическая логика - часть математики в силу того, что с методологической точки зрения она совершенно неотделима от других разделов чистой математики и имеет более-менее общее с ними происхождение. Сразу отмечу, что теоретическая информатика тоже часть математики.

Да, безусловно, она несколько изолирована от остальных частей математики. Тем не менее, перечислю некоторые связи. Есть наука про теории типов, их категорные и топологические семантики. Есть множественные связи с теоретической информатикой и теорией множеств (но для людей не считающих логику это видимо неинтересно, т.к. эти разделы математикой скорее всего они также не считают). Есть ряд связей между комбинаторикой (а может и это не математика?) и логикой - это в основном либо обобщения в логических терминах комбинаторных задач, либо логический анализ аксиом нужных для доказательства комбинаторных теорем. Есть упомянутые выше приложения теории моделей, правда, с логической точки зрения, они, как правило, весьма элементарны и находятся где-то на уровне применения теоремы о компактности. Есть ряд пересечений с универсальной алгеброй (либо со стороны теории моделей, либо со стороны неклассических логик).
Аноним 05/06/16 Вск 21:13:24 #258 №367496 
>>367494
Неплохой уход от ответа.
Аноним 05/06/16 Вск 21:20:14 #259 №367497 
>>367494
>Тогда просто докажи, что он существует.
>КО-Ко-КО, КУДКУДАХ
Аноним 05/06/16 Вск 21:45:05 #260 №367498 
>>367497
>>367496
Что не понятно?
Аноним 05/06/16 Вск 21:47:54 #261 №367499 
>>367498
А что тут должно быть понятно, кроме того, что ты глупо пошутил?
Аноним 05/06/16 Вск 21:52:50 #262 №367501 
14651527709240.png
Анон, есть две функции, точнее, их графики, какая из них больше? Зеленая или красная?
Аноним 05/06/16 Вск 21:53:37 #263 №367502 
>>367501
Что значит больше?
Аноним 05/06/16 Вск 21:56:08 #264 №367503 
>>367502
Ну, всмысле выше. Бля, я хз, как тебе объяснить. Я просто решаю чертов интеграл, и мне нужно вычесть большую функцию из меньшей.
Аноним 05/06/16 Вск 21:57:25 #265 №367504 
>>367503
Опять инженероотстойник прорвало.
Аноним 05/06/16 Вск 21:58:38 #266 №367505 
>>367504
В каком смысле? что ты имеешь против инженеров?
Аноним 05/06/16 Вск 21:59:31 #267 №367506 
>>367499
Что тебя смущает? Это серьёзный ответ, если что.
Аноним 05/06/16 Вск 22:00:37 #268 №367507 
>>367505
Твой вопрос звучит некорректен. Или ты троллишь?
Аноним 05/06/16 Вск 22:00:43 #269 №367508 
>>367505
Я не тот анон, но ты реально даун, у тебя с элементарным мышлением проблемы.
Аноним 05/06/16 Вск 22:02:59 #270 №367509 
>>367507
>Твой вопрос звучит некорректен.
Это. Ладно, извините за беспокойство.

>>367508
Говна наверни, мамкин Лобаческий.
Аноним 05/06/16 Вск 22:04:12 #271 №367510 
>>367508
Хоть ты и даун полнейший, я попробую предположить, что ты хочешь сделать, и помочь. Мне кажется, ты хочешь посчитать площадь фигуры ограниченной зеленой и оранжевой кривыми. Для этого тебе надо найти левую точку пересечения (a) и правую (b); ответом будет интеграл от зеленой на [a, b] минус интеграл от оранжевой на [a, b].
Аноним 05/06/16 Вск 22:05:19 #272 №367511 
>>367510
Спасибо, антош, я просто выразился хуево. От души спасибо.
Аноним 05/06/16 Вск 22:24:14 #273 №367513 
>>366963 (OP)
Как себе представить четвертое измерение? Три измерения это когда три палки перпендикулярны друг другу. А как сюда приткнуть еще одну палку, чтобы она была перпендикулярна трем остальным?
Аноним 05/06/16 Вск 22:24:41 #274 №367514 
>>367506
Что значит, что меня смущает? Ты глупость пёрднул, а меня смущать должно?
Аноним 05/06/16 Вск 22:26:52 #275 №367515 
>>367513
Представь себе трехмерный объект, изменяющийся во времени. Так вот если взять какой-то отрезок времени, то это четырехмерный объект.
Аноним 05/06/16 Вск 22:28:22 #276 №367516 
>>367514
>>367506
Ну т.е. как я и сказал >>367204 нет никакой "гладкости" которая идёт с топологическим многообразием "вообще", что типа сфера и какой-то элипсоид "гладкий", а куб, икосаэдр, тетраэдр или что там ещё "не гладкие". Когда мы говорим, что дано гладкое многообразие мы имеем ввиду что берётся топологическое многообразие и говорится что значит что функция "гладкая" на нём. Это делается через задание гомеоморфизма окрестности каждой точки некоторой окрестности R^n особым образом, и гладкой функцией считается функция, которая получается гладкой "в координатах" - т.е. то что композиция с заданным гомеоморфизмом гладкая в смысле обычного анализа как функция из R^n в R^k. Поэтому говорить диффеоморфен или нет куб икосаэдру, а икосаэдр сфере бессмысленно без фиксирования этой структуры, эти многообразия топологически неотличимы - поэтому естественно требуемые гомеоморфизмы в R^n можно задать просто перенеся их со сферы после того как зафиксируем какой-то гомеоморфизм икосаэдра со сферой, и он будет диффеоморфизмом по построению тогда.
Аноним 05/06/16 Вск 22:38:06 #277 №367517 
Есть качественные видеолекции по группам Ли?
Аноним 05/06/16 Вск 22:48:55 #278 №367518 
>>367517
Если качественные - это не про качество записи посмотри лекции Смирнова в НМУ 2015 года.
Аноним 05/06/16 Вск 23:28:40 #279 №367519 
14651585201500.jpg
Аноним 05/06/16 Вск 23:47:50 #280 №367520 
>>367519
Ключевое слово "подмногообразие".
https://ru.wikipedia.org/wiki/%D0%93%D0%BB%D0%B0%D0%B4%D0%BA%D0%BE%D0%B5_%D0%BC%D0%BD%D0%BE%D0%B3%D0%BE%D0%BE%D0%B1%D1%80%D0%B0%D0%B7%D0%B8%D0%B5#.D0.9F.D0.BE.D0.B4.D0.BC.D0.BD.D0.BE.D0.B6.D0.B5.D1.81.D1.82.D0.B2.D0.B0_.D0.B8_.D0.B2.D0.BB.D0.BE.D0.B6.D0.B5.D0.BD.D0.B8.D1.8F
А откуда дровишки?
Аноним 05/06/16 Вск 23:59:51 #281 №367521 
>>367520
Арнольд, Что такое математика.
Аноним 06/06/16 Пнд 00:01:31 #282 №367522 
>>367521
Ясно.
Аноним 06/06/16 Пнд 00:03:19 #283 №367523 
>>367513
Цветом можно. От красного до синего.
Аноним 06/06/16 Пнд 00:07:01 #284 №367524 
>>367519
Сука, стиль как у Рыбникова.
Аноним 06/06/16 Пнд 00:14:09 #285 №367526 
>>367516
Это философский пердёж, а не математика. К тому же подмена понятий, мол, я такой альтернативно умный, буду все понятия переопределять как мне хочется.
Аноним 06/06/16 Пнд 00:20:12 #286 №367527 
>>367522
Но профессор действительно дебил. Все знают, что подмножества R^n имеют (или не имеют) канонические гладкие структуры, а то, что на подмножестве R^n можно ввести структуру не индуцированную из R^n - никого не ебёт.
Аноним 06/06/16 Пнд 00:52:28 #287 №367528 
>>367526
> Это философский пердёж, а не математика.
Именно что математика, покажи мне где вообще кто-то строит гомеоморфизмы между сферой и икосаэдром явно в математике.
> понятия переопределять
https://ru.wikipedia.org/wiki/%D0%93%D0%BB%D0%B0%D0%B4%D0%BA%D0%BE%D0%B5_%D0%BC%D0%BD%D0%BE%D0%B3%D0%BE%D0%BE%D0%B1%D1%80%D0%B0%D0%B7%D0%B8%D0%B5
Топологическое многообразие X, наделенное C^k -структурой, называется C^k -гладким многообразием.
Аноним 06/06/16 Пнд 04:06:28 #288 №367531 
>>367527
дебил уровня двача, где каждый кого угодно может назвать дебилом. как "крупный математик" - по оценке самого обвиняющего - может быть одновременно дебилом?
Аноним 06/06/16 Пнд 04:57:12 #289 №367532 
>>367531
Ты что, дебил?
Аноним 06/06/16 Пнд 05:19:18 #290 №367534 
>>367532
У тебя вопросы какие-то дебильные, честное слово.
Аноним 06/06/16 Пнд 06:53:12 #291 №367537 
>>367532
>>367534
Вы только посмотрите на эти дебилов!
Аноним 06/06/16 Пнд 09:16:17 #292 №367543 
14651937780460.jpg
Дебилы, блядь.
Аноним 06/06/16 Пнд 09:18:10 #293 №367544 
Планирую обучаться по программе Вербицкого - http://imperium.lenin.ru/~verbit/MATH/programma.html
Какие подводные?
Аноним 06/06/16 Пнд 09:19:39 #294 №367545 
>>367544
Не осилишь с вероятностью 99.9%
Аноним 06/06/16 Пнд 09:36:01 #295 №367546 
>>367545
>146%
Поправил чутка
Аноним 06/06/16 Пнд 10:26:24 #296 №367547 
>>367544
Для большинства людей прочтение даже одной книги по математике является подвигом. Тебе придётся прочитать сотни книг и статей.
Аноним 06/06/16 Пнд 11:17:54 #297 №367551 
>>367544
Подводные камни в том, что это не программа, а список тем. Они не упорядочены, а просто скинуты в одну корзину.
Я изредка посматриваю и наблюдаю как количество непонятных слов падает.
Аноним 06/06/16 Пнд 11:18:20 #298 №367552 
>>367551
Они упорядочены.
Аноним 06/06/16 Пнд 11:39:23 #299 №367557 
>>367546
>>367545
Почему?
Аноним 06/06/16 Пнд 12:36:59 #300 №367563 
>>367557
Сложно.
Аноним 06/06/16 Пнд 13:18:17 #301 №367564 
Поясните за интеграл Лебега. Как его вычислять-то? Я так понимаю, он дает только более простые доказательства результатов (например, более простые условия когда можно почленно интегрировать ряд), а когда дело доходит до вычислений, то мы можем реально вычислить интеграл от функции только если она "хорошая", то есть, к примеру, интегрируемая по Риману.
Аноним 06/06/16 Пнд 13:32:57 #302 №367569 
>>367495
Двачую адеквата
Аноним 06/06/16 Пнд 13:39:08 #303 №367570 
>>367563
В чём сложность?
Аноним 06/06/16 Пнд 13:54:19 #304 №367573 
http://ium.mccme.ru/f04/experimental.html
а вот про этот курс что скажете? стоит по нему заниматься например параллельно с Давидовичем? (Давидовича я в любом случае не брошу, он охуенный)
Аноним 06/06/16 Пнд 14:33:53 #305 №367578 
14652128340670.png
Братишки, как решать задачи такого рода?
Аноним 06/06/16 Пнд 14:35:06 #306 №367579 
>>367578
Использовать признак сравнения.
Аноним 06/06/16 Пнд 14:40:57 #307 №367581 
>>367579
Да ладно?! С каким интегралом сравнивать?
Аноним 06/06/16 Пнд 14:53:39 #308 №367583 
Посаны, чет я туплю. Пусть f: X -> Y, X и Y - топологические пространства. Верно ли, что f непрерывна в p <=> (pn -> p => f(pn) -> f(p))? Очевидно, что => верно, но верно ли <=? Для метрических пространств это верно, потому что если предположить, что f претерпевает разрыв в p, можно брать pn из окрестности радиуса 1/n и получить противоречие. А вот для топологических я хз.
Аноним 06/06/16 Пнд 14:55:34 #309 №367584 
>>367200
>Физика - это раздел математики
у нас на физфаке за такое убивают нахер
Аноним 06/06/16 Пнд 14:56:37 #310 №367585 
>>367584
Ну и хуле в мат-треде делаешь? Иди в свой загон! Тут физика лишь часть математики.
Аноним 06/06/16 Пнд 15:43:46 #311 №367592 
>>367585
А у нас нет загона, нет физ. треда.
Аноним 06/06/16 Пнд 16:43:42 #312 №367600 
>>367592
Создай тогда. Лол. Будешь ОП-ом, солидная должность.
Аноним 06/06/16 Пнд 17:02:36 #313 №367603 
>>367583
Все, уже не надо. <= верно, если X first-countable.
Аноним 06/06/16 Пнд 17:06:28 #314 №367604 
>>367600
Этот человек лжёт. Быть ОПом - зашквар. Это как петух в тюрьме.
Аноним 06/06/16 Пнд 17:07:40 #315 №367605 
>>367604
Ты просто не понял сарказма.
- мимо этот человек
Аноним 06/06/16 Пнд 17:08:16 #316 №367606 
>>367605
Объясни сарказм.
Аноним 06/06/16 Пнд 17:08:55 #317 №367607 
>>367606
Объяснил, проверяй.
Аноним 06/06/16 Пнд 17:09:51 #318 №367608 
>>367607
Где?
Аноним 06/06/16 Пнд 17:11:31 #319 №367609 
>>367608
За щекой у тебя, внимательнее посмотри.
Аноним 06/06/16 Пнд 17:13:00 #320 №367610 
>>367609
Сорри, у меня за щекой только копчик твоего деда.
Аноним 06/06/16 Пнд 17:15:27 #321 №367611 
>>367610
Просто ты не знаешь определение N. Поэтому, ты не можешь нормально считать. Определи N, после ещё раз посмотри.
P.S.
Определение N тоже у тебя за щекой.
Аноним 06/06/16 Пнд 17:18:48 #322 №367612 
>>367611
Мы нуждаемся в новой науке - замене математики. Математика умерла. Она мертва. Давайте создавать новое знание.
Аноним 06/06/16 Пнд 17:21:11 #323 №367613 
>>367612
Да. Ты читаешь мои мысли. Сделаем новую математику на основе программы Вербицкого! А Вербицкого сделаем пророком и будем внимать каждому его слову!
Аноним 06/06/16 Пнд 17:26:42 #324 №367615 
>>367613
Согласен с Вами, друг.
Аноним 06/06/16 Пнд 18:06:39 #325 №367625 
>>367545
Потому что дебил я? Или вербит дебил? Давайте конкретно говорить, отталкиваясь от дебилов.
Аноним 06/06/16 Пнд 18:09:23 #326 №367626 
>>367625
Оба хороши.
Аноним 06/06/16 Пнд 18:10:16 #327 №367627 
>>367604
Откуда математики так подкованы в тюремном жаргоне? Прямо сыпят им в каждом сообщении. Петухи маня два стула. Это только для российской интеллигенции свойственно или это мировая тенденция?
Аноним 06/06/16 Пнд 18:10:22 #328 №367628 
>>367625
Может, потому что одному сложной математикой сложно заниматься.
Аноним 06/06/16 Пнд 18:11:31 #329 №367629 
>>367627
Это двач. Тут каждый второй так говорит.
Аноним 06/06/16 Пнд 18:34:53 #330 №367630 
>>367625
Программа абсолютно анреал для студентов. И, на самом деле, слишком специальная, всем математикам нужно знать только первые два курса и некоторые темы из третьего, дальше, специализация именно в вербитоговне.
Аноним 06/06/16 Пнд 18:39:39 #331 №367631 
>>367630
То есть дебил все таки ОН. А не Я. Для кого тогда этот список, тралит он так?
Аноним 06/06/16 Пнд 18:46:40 #332 №367632 
>>367631
Очевидно для тех, кто хочет заниматься теми же областями, что и Вербит.
> Математика лишь постольку интересна, поскольку она связана со струнной теорией; это базовое предположение, которое я не хочу сейчас обсуждать. Релевантность для физики это единственный критерий, который у нас остался; а почти вся математика, относящаяся к физике, относится к струнной геометрии. Этот тезис хорошо подтверждается наблюдением, приведенным выше: (почти) все интересные идеи последних 20 лет связаны с физикой струн.
Аноним 06/06/16 Пнд 19:14:06 #333 №367636 
>>367573
пацаны, а про это что скажите?
или Вербицкий = гавно?
Аноним 06/06/16 Пнд 19:28:57 #334 №367638 
14652305376420.jpg
Горит пересдача и остается один очень важный и совершенно нерасписанный билет. Пожалуйста, подскажите где можно посмотреть чтобы был дотошно откомментирован каждый шаг, или может кто сам знает как расписать через уравнение Эйлера-Лагранжа:
1) принцип Дирихле
2) обобщенный принцип Дирихле
3) нелинейное уравнение Пуассона
4) минимальная поверхность
Аноним 06/06/16 Пнд 19:29:30 #335 №367639 
>>367632
Вербит ни разу не авторитет, что все так на него равняются. Есть и были гораздо более сильные и глубокие математики, просто они не постят в жж.
Аноним 06/06/16 Пнд 19:43:39 #336 №367641 
14652314193720.jpg
>>367578
Бамп вопросу, как подбирать интеграл для сравнения?
Аноним 06/06/16 Пнд 19:45:12 #337 №367642 
>>367641
Так же как с рядом.
Аноним 06/06/16 Пнд 19:49:19 #338 №367643 
>>367642
Но я же не знаю степень лямбда.
Аноним 06/06/16 Пнд 19:50:08 #339 №367644 
>>367643
Тебе и надо ее так подобрать чтобы ряд сходился.
Аноним 06/06/16 Пнд 19:54:58 #340 №367645 
>>367537
этиХ, а не эти. дебил.
Аноним 06/06/16 Пнд 20:14:36 #341 №367649 
>>367644
Ряд подынтегральной функции в данном примере? То есть мне не нужно подбирать еще одну функцию и искать предел их отношений?
Буду очень благодарен если напишешь пошаговое пояснение на этом примере. Я чрезвычайно туплю и уже запутался.
Аноним 06/06/16 Пнд 20:22:03 #342 №367652 
>>367578
Двач нынче обмельчал. Очевидно, что твоя подинтегральная функция на бесконечности ~ x^(λ - 2).
Очевидно, что твоя функция должна убывать быстрее чем 1/n, чтобы интеграл сходился. Вспоминаем, что ряд Дирихле Σ 1/n^a сходится при a > 1. Т.е для нашего случая 2 - λ > 1 => λ < 1.
Аноним 06/06/16 Пнд 20:28:25 #343 №367653 
>>367639
В этом и проблема. Через свой журнальчик и дешевый эпатаж посредством страпона навязывает свои понятия о том, "что такое математика и как ее изучать". Очередной нуб, перечитав вербита, решит что гамалогии и тп и есть "правильная современная математика". А на самом деле это какой-то узкий набор тем из миллиона различных направлений.
Аноним 06/06/16 Пнд 20:33:12 #344 №367654 
>>367652
дирихле это современная математика или картофан?
Аноним 06/06/16 Пнд 20:34:19 #345 №367655 
Есть у кого дуфуры интуры матфизика том 1 за авторство Франка и Мизеса, а то уже весь инет перерыл, ток второй том нашёл?
Аноним 06/06/16 Пнд 20:34:33 #346 №367656 
>>367654
Это фамилия.
Аноним 06/06/16 Пнд 20:36:14 #347 №367657 
>>367655
нет, сорян, я только знаю есть кажется земля франка и мизеса. вроде там дубак и льды.
Аноним 06/06/16 Пнд 20:39:03 #348 №367659 
>>367528
Ну ты дальше прочитай. Какие должны быть отображения. Ты же спорол хуйню, мол у тебя карты состоят из композиций гомео на сферу, а потом там диффео.
Аноним 06/06/16 Пнд 20:41:34 #349 №367661 
Вроде понял, все легче чем я думал. Спасибо.
Аноним 06/06/16 Пнд 20:41:53 #350 №367662 
>>367661
->>367652
Аноним 06/06/16 Пнд 20:44:07 #351 №367663 
>>367639
>Вербит ни разу не авторитет
Всего-то был одним из четырёх делегатов от России на недавнем международном математическом конгрессе. Подумаешь. У меня эти делегаты каждую ночь под окнами бегают.
Аноним 06/06/16 Пнд 21:02:29 #352 №367666 
14652361491270.jpg
>>367564
Бамп вопросу
Аноним 06/06/16 Пнд 21:17:53 #353 №367667 
>>367564
>>367666
Лучше онтологически разделять определение интеграла и методы его вычислений. Для некоторых хороших классов функций удаётся подогнать годные методы вычисления, для некоторых - нет, соу ват?
Аноним 06/06/16 Пнд 21:17:59 #354 №367668 
14652370793680.png
>>366963 (OP)
Элементарная геометрия. Как доказать, что угол y меньше угла b?
Аноним 06/06/16 Пнд 21:23:54 #355 №367669 
>>367663
конгресс по гамологиям был поди. тем более там небось тоже разборчивые - обычных математиков переели уже, привези-ка им этакова некаквсе, с журнальчиком и страпончиком.
Аноним 06/06/16 Пнд 21:24:34 #356 №367670 
>>367667
Это я к тому, что нужно ли мне изучать интеграл Римана? Вот я читал Рудина, дошел до интегралов, а там даже не интеграл Римана, а интеграл Римана-Стильтеса, то есть вообще какой-то упоротый. Я решил изучить нормальный интеграл Лебега, а потом дальше читать Рудина, но доказывать все теоремы оттуда с помощью интеграла Лебега. Это норм подход?
Аноним 06/06/16 Пнд 21:29:38 #357 №367671 
>>367668
Блядская иисусня, я наконец-то понял! Угол "b" больше угла "y" вследствие свойства, что внешний угол треугольника всегда больше любого из противоположных ему углов в этом треугольнике.
Аноним 06/06/16 Пнд 21:30:12 #358 №367672 
>>367670
Норм подход, так и делай. Могу посоветовать ещё Львовский "Лекции по математическому анализу", там вводится так называемый "Интеграл Коши", который потом всё равно заменяется интегралом Лебега.
Аноним 06/06/16 Пнд 21:33:34 #359 №367673 
>>367670
Норм. Только вот с "доказывать все теоремы оттуда с помощью интеграла Лебега" у тебя будут проблемы. У Рудина Риман-Стилтьес не зря.
Аноним 06/06/16 Пнд 21:34:15 #360 №367674 
>>367673
А зачем он там, кстати?
другой ананас
Аноним 06/06/16 Пнд 21:37:15 #361 №367676 
>>367672
Львовского тоже читаю, но там многие вещи пропускаются как сами собой разумеющиеся и как-то бегло все. Скачал еще пару книг, которые советовали на MSE и выбрал Folland: Real analysis modern techniques, там вроде довольно подробно все расписывается. На Львовского посматриваю в качестве ориентира.
Аноним 06/06/16 Пнд 22:12:28 #362 №367678 
Двачеананасы, почему математика такое говно ебаное?
Почему вместо того чтобы сказать что ряд сходится если общий член ряда приближается к нулю мне начинают задвигать про сходимость по коши для сердца и души, по даламберу, по ньютону, по эйнштейну, по минковскому, по гауссу, по небу и по аллаху? Зачем все это? Кому нужна эта жопоебля?
Аноним 06/06/16 Пнд 22:14:45 #363 №367680 
>>367678
Если тебе это не нужно - то и не трогай это, лал.
Аноним 06/06/16 Пнд 22:16:11 #364 №367681 
>>367680
>Задал четкий и внятный вопрос
>АЗАЗА ПРИПЕКЛОУ ТАК ПРИПЕКЛОУ, АХАХАХ ЛАЛКА
Аноним 06/06/16 Пнд 22:17:51 #365 №367682 
>>367681
При чём тут это? Если тебе интересен некоторый кусок математического знания, то можешь изучать его, если неинтересен - то и ненужно. Теория расходящихся рядов весьма специфическое знание, и вполне можно быть хорошим математиком и ничего об этом не знать. Или ты про что?
Аноним 06/06/16 Пнд 22:22:12 #366 №367683 
>>367682
Я про то, почему меня сразу грузить надо кошелками с дамблдорами? Почему просто не сказать что ряд сходится если он стремится к нулю, прицепить к этому притчу про производные и натуральный ряд, через который проходит сам ряд, и только потом начать уточнять что мол "ряд может и не сходится, даже если он в передел равен нулю, что доказал нянькин-бабский в соавторстве с ногой-макарониной в своей теории кранца начиханца в супрематическом году от дня рождения тети сраки" и потом уже начинать грузить коши и даламбером.
Аноним 06/06/16 Пнд 22:25:31 #367 №367684 
>>367682
То же самое и с интегралом. Сперва можно было обосновать его как подсчет квадратиков на листочке в клеточку, потом сказать что квадратики считать тяжело и на производные съехать, а потом съехать уже на интегралы.
Аноним 06/06/16 Пнд 22:26:58 #368 №367685 
И да, всякому ли математическому объекту/функции/чему угодно из математики можно сопоставить чертеж на плоскости/объеме/многомерном пространстве?
Аноним 06/06/16 Пнд 22:32:54 #369 №367688 
>>367685
Нет.
Аноним 06/06/16 Пнд 22:34:58 #370 №367689 
>>367688
Пример нета будет?
Аноним 06/06/16 Пнд 22:36:42 #371 №367690 
>>367689
Бесконечномерная сфера.
Аноним 06/06/16 Пнд 22:39:28 #372 №367691 
>>367690
ПроЭкцией переносится на плоскость, небо, аллаха, проекцией на пятое измерение, которое проекцией на четвертое, проекцией на третье, проекцие на второе измерение экрана монитора комьпютера.
Аноним 06/06/16 Пнд 22:40:38 #373 №367692 
>>367691
Окей. Сильно недостижимый кардинал.
Аноним 06/06/16 Пнд 22:43:02 #374 №367693 
>>367692
Собрать бесконечное количество мушкетеров, чтобы они каждый кардинал лично пересчитали и сказали какой больше, нарисовав на кардинальной плоскости, в масштабе, какой размер у какого кардинала.
Аноним 06/06/16 Пнд 22:44:37 #375 №367694 
>>367693
Шиза какая-то.
Аноним 06/06/16 Пнд 22:47:56 #376 №367696 
>>367694
Ну смотри, если у нас вселенная бесконечная, то множество мыслящих существ, получившихся в результате эволюции, будет бесконечно.
Так же само будет бесконечно множество мыслящих существ получившихся в результате больцмановского мозга, тобишь из говна и палок напрямую.
И то и то бесконечно, но вероятность, а следовательно и кардинал, того что на какой то занюханной планете появятся лысые макаки сильно больше вероятности того что где то в далекой галактике кучка газа соберется в мыслящий самоосознающий конструктив.
Аноним 06/06/16 Пнд 22:50:30 #377 №367698 
>>367694
Бесконечности разных размерностей, во.
И если представить себе что у нас есть кардинальное пространство, содержащееся на балансе его королевского величества, то на ней можно обозначить что множество А меньше множества Бэ в корень из двух раз.
Аноним 06/06/16 Пнд 22:50:40 #378 №367699 
>>367696
Если ты самоосознающий и мыслящий конструктив из кучки газа, то мой совет - пиздуй с саентачей, тут всё, что не понимают хуесосят и называют вербитоговном.
Аноним 06/06/16 Пнд 22:51:01 #379 №367700 
>>367694
Тобишь, и то и то бесконечность, но вот эта бесконечность больше.
Аноним 06/06/16 Пнд 22:52:10 #380 №367701 
>>367699
https://en.wikipedia.org/wiki/Boltzmann_brain
>A Boltzmann brain is a hypothesized self aware entity which arises due to random fluctuations out of a state of chaos.

Это вам не бурбакистов читать, да.
Аноним 06/06/16 Пнд 22:53:16 #381 №367702 
>>367701
Ну ты смотри, моё дело предупредить.
Аноним 06/06/16 Пнд 22:56:45 #382 №367703 
>>367678
> Почему вместо того чтобы сказать что ряд сходится если общий член ряда приближается к нулю
Потому что это неверно. Самый простой пример: гармонический ряд.
Аноним 06/06/16 Пнд 23:03:17 #383 №367704 
>>367678
Научись отличать необходимое условие от достаточного условия. Сходимость общего члена ряда к нулю - необходимое условие сходимости ряда. Но отнюдь не достаточное.
Аноним 06/06/16 Пнд 23:05:25 #384 №367705 
>>367703
>>367704
Гармоничный ряд это не вообще все ряды.
Тут как в ебле все, сперва бухой в говно пытаешься ебюаться, потом без алкоголя, потом не крокодилов, а потом и топ моделей.
А мне тут предлагают, девственнику неопытному, сразу начать ебать джесику альбу, или кто там щас популярен, и удивляются почему у меня полшестого.
Аноним 06/06/16 Пнд 23:07:10 #385 №367706 
>>367705
Так-то джессика альба норм, я не знаю почему у тебя полшестого.
мимо ебался один раз в жизни по пьяне
Аноним 06/06/16 Пнд 23:08:54 #386 №367707 
>>367705
Ты походу совсем тупой, с элементарным мышлением проблемы, математика тут ни при чем.
Аноним 06/06/16 Пнд 23:12:01 #387 №367708 
>>367707
Еще раз, для контуженных жизнью даунов.
Да, сходимость к нулю ряда джостаточное, но не обязательное условие сходимости, гармонические ряды это опровергают.
НО, гармонические ряды это специфические ряды. Для какого то ряда икс куб или два икс делить на три икс увидеть сходимость, есть она или нет, можно только по этому правилу. И вот уже с этого правила извлекать исключения и уточнения.
Аноним 06/06/16 Пнд 23:19:33 #388 №367709 
>>367708
> И вот уже с этого правила извлекать исключения и уточнения.
Ну так и делают. Когда видят ряд, сначала проверяют, стремится ли общий член к 0. Если не стремится, ряд расходится. Если стремится, то нужно использовать другие инструменты, чтобы выяснить, сходится ли он. В чем проблема?
Аноним 06/06/16 Пнд 23:20:34 #389 №367711 
>>367708
>сходимость к нулю ряда джостаточное, но не обязательное условие сходимости
Ты в состоянии прочитать пост, на который отвечаешь? Сходимость общего члена ряда к нулю - необходимое условие сходимости ряда. Но отнюдь не достаточное.
Аноним 07/06/16 Втр 00:25:53 #390 №367714 
>>367659
> Ну ты дальше прочитай. Какие должны быть отображения
Какие? Если ты про то, что там про непрерывные функции ничего не сказано, а только ебля с атласами - ну так это же русская википедия. Ты прав в том что есть куда более человеческое определение гладкого многообразия через пучки, эквивалентное данному. Можно посмотреть например в английской википедии https://en.wikipedia.org/wiki/Differentiable_manifold#Structure_sheaf
Когда я тут объяснял интуицию всего этого дела, я конечно использовал то что о гладком многообразии разумно думать таким образом, а не через какую-то муть с картами и атласами. Гладкость отображения в первом смысле эквивалетна тому что гладкие функции переводятся в гладкие - достаточно очевидно после первой минуты размышлений.
Но, даже если ты против, это всё вообще можно считать лирическим отступлением. То что при переносе структуры с помощью гомеоморфизма со сферой этот гомеоморфизм оказывается гладким и биективным - это в общем-то тавтология, хоть в первом определении, хоть во втором. Это то же самое когда ты говоришь что, взяв в качестве топологии X прообразы открытых множеств из Y при отображении f из X в Y, это отображение f у тебя получается непрерывным по построению.
Аноним 07/06/16 Втр 00:53:18 #391 №367715 
14652499990500.png
>>367676
Бля, вроде ничего сложного, но пиздос абстрактно.
Аноним 07/06/16 Втр 01:02:43 #392 №367716 
>>367715
Можешь Тао навернуть, он лучше переваривается.
Аноним 07/06/16 Втр 01:19:34 #393 №367718 
математики обсуждают еблю
Аноним 07/06/16 Втр 01:58:52 #394 №367726 
>>367718
Твоей мамки
Аноним 07/06/16 Втр 02:23:57 #395 №367728 
Единственный математик, который подкован в вопросе ебли - это Миша Вербицкий. Пишет, что ощущения интересные.
Аноним 07/06/16 Втр 04:46:54 #396 №367730 
>>367685
Нет. Гугли функцию Дирихле.
Аноним 07/06/16 Втр 04:49:45 #397 №367731 
>>367728
Кек.
Аноним 07/06/16 Втр 08:23:55 #398 №367739 
>>367714
Мне не понравилась идея называть гладкими картами композиции из гомео на сферу и диффео на сфере. Проблема в том, что всё равно придётся отображать сами точки икосаэдра, включая "углы". Либо тогда надо слишком много всего "переопределять"
Аноним 07/06/16 Втр 09:26:03 #399 №367747 
>>367711
>>367709
Я перепутал понятия. В час ночи не стыдно, да.

И я имел ввиду чтобы сперва пояснить что такое за хуйня этот ряд, а потом уже на этом начать строить все остальные выводы из нихъ.
Я имею ввижу что изложение в учебниках очень тупое.
Аноним 07/06/16 Втр 10:01:39 #400 №367752 
>>367730
Но ведь можно тупо прямые по всем целым числам и сказать что это график такой функции.
Аноним 07/06/16 Втр 10:15:40 #401 №367754 
>>367752
Нельзя.
Ты пропустишь бесконечно много рациональных.
Аноним 07/06/16 Втр 10:18:21 #402 №367755 
>>367754
Опять перепутал. По всем рациональным имел ввиду. Получится настолько густая частота линий, что нам она будет казаться плоскостью декартовой.
Или если мы на числовой прямой обозначаем то той же прямой.
Аноним 07/06/16 Втр 10:27:02 #403 №367756 
>>367755
Ну тогда как пример - какая-нибудь фрактальная линия. Мы сможем увидеть её график только приближенно, и чем больше будем приближать тем сложнее будет становится участок приближения.
Аноним 07/06/16 Втр 10:33:51 #404 №367757 
>>367756
Че так? Фрактал он и в африке фрактал. Как ты его ни приближай он все равно будет однообразной загогулиной, бесконечной загогуленности но конечной длинны.
Аноним 07/06/16 Втр 10:35:04 #405 №367758 
>>367757
Но график-то ты не сможешь начертить, верно?
Аноним 07/06/16 Втр 10:39:11 #406 №367760 
>>367747
> И я имел ввиду чтобы сперва пояснить что такое за хуйня этот ряд
Ты реально олигофрен. Покажи мне учебник, в котором не дается определение ряда, но даются признаки сходимости.
Аноним 07/06/16 Втр 11:15:52 #407 №367762 
>>367758
Все математические объекты можно отобразить. Если математический объект нельзя отобразить, значит о нем нельзя сказать, и значит такой объект существует только в манямирке ощущающего его математика, а остальные о нем никогда не узнают.
Детский же вопрос о том, все ли можно нарисовать - да, все, если не учитывать силы, которые потратишь на придумывание способа отображения и доказывание остальным, что этот способ - верный.
Аноним 07/06/16 Втр 11:29:35 #408 №367764 
14652881757770.png
Мне кажется, в википедии ошибка. Как исправить?
https://en.wikipedia.org/wiki/Order_topology
Пикрелейтед правильное определение.
Разница будет, например, если вводить order topology на extended real numbers (что я пытался сделать и долго тупил).
Аноним 07/06/16 Втр 12:10:34 #409 №367765 
>>367764
А ничего, что по твоему определению порядковая топология на вещественных - это дискретная топология (для всякого a, пересечение (a-1,a] и [a,a+1) - это сама точка a)?
Аноним 07/06/16 Втр 12:14:20 #410 №367766 
>>367765
Да, в самом деле, - не прав. У тебя X - это множество на котором задан порядок.
Но думаю, что лучше определять, как топологию с предбазой из всех открытых лучей.
Аноним 07/06/16 Втр 13:11:06 #411 №367773 
>>367765
Всё нормально по его определению, полинтервалы разрешается брать когда конец полуинтервала - наименьший или наибольший соответственно.
Аноним 07/06/16 Втр 13:36:26 #412 №367777 
>>367760
Любой открой и посмотри.

>>367758
Полностью не смогу, примерно смогу.
Аноним 07/06/16 Втр 14:29:47 #413 №367783 
>>367739
Так нет топологически никаких углов. А координаты, которые должны быть гладкими, ты задаёшь именно топологически через гомеоморфизм с R^n. И если ты это делаешь через сферу, все отображения переклейки у тебя получается в точности такими как на сфере, т.е. гладкие.
Аноним 07/06/16 Втр 16:41:11 #414 №367804 
>>367747
это не ты доказывал что trollface pi = 4? где-то этот пидор тут сидит. опять по целам числам прямые строятся.
Аноним 07/06/16 Втр 17:44:28 #415 №367818 
А кто именно книги бурбаки писал?
Аноним 07/06/16 Втр 17:46:01 #416 №367819 
>>367818
Я.
Аноним 07/06/16 Втр 18:26:46 #417 №367822 
14653132070030.jpg
>>367818
Аноним 07/06/16 Втр 18:31:00 #418 №367824 
>>367822
Молокососы с манией величия, короче говоря.
Аноним 07/06/16 Втр 18:56:11 #419 №367828 
>>367824
Умри.
Аноним 07/06/16 Втр 18:56:31 #420 №367829 
>>367824
_Кек_
Аноним 07/06/16 Втр 18:59:55 #421 №367830 
>>367824
да, все лица двачерские. их травили в шкалке похоже. и вот они отомстили. здесь бы они наверняка друг друга бы хуесосили и N определяли.
Аноним 07/06/16 Втр 19:08:39 #422 №367831 
>>367830
Хуй соси.
Аноним 07/06/16 Втр 19:20:44 #423 №367833 
>>367783
Что-то я тебя плохо понимаю, слишком размыто. Дай-ка лучше ссылку на что-нибудь похожее на то, что ты имеешь ввиду.
Аноним 07/06/16 Втр 19:51:24 #424 №367841 
Планирую стать Зориче-ёбом. Какие подводные камни?
Аноним 07/06/16 Втр 19:57:00 #425 №367842 
>>367841
Есть интеграл Римана, который позже не заменяется на интеграл Лебега, бывают размытые задания. Во втором томе есть дебильные главы, вроде равномерной сходимости. Так-то учебник годнота лютейшая.
мимо зоричеёб
Аноним 07/06/16 Втр 20:05:03 #426 №367843 
>>367842
> дебильные главы, вроде равномерной сходимости
Поясни за это. Как без нее обосновать почленное интегрирование ряда?
Аноним 07/06/16 Втр 20:16:28 #427 №367848 
>>367843
Monotone/bounded convergence theorem для общих пространств с мерой.
Аноним 07/06/16 Втр 20:24:46 #428 №367854 
Аноны, посоветуйте учебники для повторения всей школьной алгебры и геометрии для поступления, а то по треду не понятно, хуй всё-таки оп или нет.
Аноним 07/06/16 Втр 20:29:38 #429 №367857 
>>367833
Вот например
http://math.stackexchange.com/questions/106494/is-a-cube-a-smooth-manifold
Аноним 07/06/16 Втр 20:34:45 #430 №367861 
>>367842
В ангельском переводе он нормально читается?
Аноним 07/06/16 Втр 20:36:03 #431 №367862 
>>367861
Не знаю, не было никогда желания читать на ангельском то, что в оригинале на русише. Это же аллогично.
Аноним 07/06/16 Втр 20:40:28 #432 №367863 
>>367862
Разве? Ведь все современные мат. работы на ангельском, а держать в уме два пакета терминов - русские определения из литературы и аналогичные им английские для понимания современных статей, как раз-таки аллогично. Лучше всё сразу на английском учить.
Аноним 07/06/16 Втр 20:40:32 #433 №367864 
>>367804
Кстати, а в чем на том пике ошибка? То, что к негладкой функции предельный переход применяют?

Мимо-нематематик
Аноним 07/06/16 Втр 20:40:59 #434 №367865 
>>367854
шень алгебра. шень геометрия
Аноним 07/06/16 Втр 20:45:12 #435 №367867 
>>367864
Там строится последовательность линий, потом на этих линиях каким-то таинственным образом совершается магический переход, и далее объявляется, что пределом является окружность. Хотя на самом деле, так как топология на множестве линий не указана, операция предельного перехода не определена.
Аноним 07/06/16 Втр 20:48:17 #436 №367869 
>>367867
А каким образом указывается топология? Как должна выглядеть та картинка, чтобы быть правильной?
Аноним 07/06/16 Втр 20:49:55 #437 №367870 
>>367867
Очевидно, что там топология индуцированная метрикой Хаусдорфа.
Аноним 07/06/16 Втр 21:00:13 #438 №367873 
>>367869
>>367864
Если очень просто на пальцах. То чтобы говорить, что одна кривая сходится к другой, мы должны выбрать модель сходимости. В одной модели сходимости (которая называется "метрикой Хаусдорфа", например) последовательность ступенчатых функций будет стремится к окружности. Но это не означает, что последовательность длин будет стремится к длине предельной кривой.
В других модельях (например в C^(1)) последовательность тех ступенчатых штук вообще не будет сходится.
И да, если тебе будут затирать что-то про "бесконечно малые" ступеньки - бей в ебало. Классическая математика построена на теоретико-множественном формализме, что означает ровно то, что кривую мы задать можем только тогда, когда сможем ответить на вопрос - какие точки принадлежат ей, а какие нет (по модулю некоторых логических тонкостей), естественно, координаты ни одной нетривиальной точки у этой "кривой с бесконечно малыми ступеньками" никто не назовёт.
Аноним 07/06/16 Втр 21:08:33 #439 №367878 
>>367873
Спасибо за пояснение. А если мы определили такую кривую со ступеньками, например, по Хаусдорфу, то как определить к чему сходится сумма их "длин"? Это вообще реальная задача?
Аноним 07/06/16 Втр 21:08:56 #440 №367879 
>>367878
Их, то есть ступенек.
Аноним 07/06/16 Втр 21:09:29 #441 №367880 
>>367878
Я не очень понял. Не мог бы ты сформулировать вопрос как-нибудь ещё?
Аноним 07/06/16 Втр 21:12:28 #442 №367881 DELETED
>>367878
ХАХАХАХАХАХАЗАХАХАХПХПЖАХВХАХА
ЛОЛ
Аноним 07/06/16 Втр 21:14:30 #443 №367882 
>>367880
Ну вот у тебя (или это не твой пост?) написано:
>В одной модели сходимости (которая называется "метрикой Хаусдорфа", например) последовательность ступенчатых функций будет стремится к окружности. Но это не означает, что последовательность длин будет стремится к длине предельной кривой.
Вот в этой модели, если последовательность длин (я так понимаю это длины ступенек?) не обязательно стремится к длине предельной кривой (это, я так понял, окружность?), то как понять когда она будет стремиться (если это вообще возможно), а когда нет? И к чему в общем случае стремится такая последовательность длин?
Аноним 07/06/16 Втр 21:16:44 #444 №367883 DELETED
>>367882
БЛЯДЬ
ХАХАХАРХААХАХА
ХАХАХАХАЗВХАХПВХАХАХАХА
ХАХАХААХАХАХПХАХАХАХАХАХАХАХА
Аноним 07/06/16 Втр 21:18:43 #445 №367885 
>>367882
>я так понимаю это длины ступенек?
Да.
>о, я так понял, окружность?
Да.
>то как понять когда она будет стремиться (если это вообще возможно), а когда нет?
Для дифференцируемых кривых достаточное условие - сходимость в C^(1) смысле. То есть сходится должны не только кривые, но и их производные.
> И к чему в общем случае стремится такая последовательность длин?
Может ни к чему не стремится, а может стремится к какому-то числу которое смысла особо не имеет. Например в случае ступенчатых функций она стремится к 4 (потому как каждый элемент последовательности равен 4).
Аноним 07/06/16 Втр 21:22:42 #446 №367886 
>>367885
А где можно про это почитать? Лучше назови конкретную главу книжки, если можно.
Аноним 07/06/16 Втр 21:24:31 #447 №367887 
>>367885
Спасибо. То есть с длиной окружности такая последовательность никак не связана?
Аноним 07/06/16 Втр 21:24:53 #448 №367888 DELETED
>>367886
Ты лох что ли?
>>367885
А ты?
Аноним 07/06/16 Втр 21:35:16 #449 №367889 
>>367865
Окей, спасибо, а сборники задач не посоветуешь, пожалуйста?
Аноним 07/06/16 Втр 21:36:28 #450 №367890 
>>367886
В общем случае нет (кроме того, что длина предельной <= lim inf длин допредельных), откуда можно получить, что pi < 4. Но опять же, при некоторых хороших условиях она может непосредственно стремится к длине предельной кривой.
>>367886
Это очень специфический вопрос. Во-первых всё-таки классически рассматривают не кривые, а функции с которыми связаны их графики - функциональные кривые.
Про модели сходимости функций можно почитать в блоге Тао https://terrytao.wordpress.com/2010/10/02/245a-notes-4-modes-of-convergence/ (или в его книжке "an introduction to measure theory" или как-то так)

Про то, почему функционал длины полунепрерывен снизу на непрерывно дифференцируемых функциях с нормой ||f||=max{||f||,||f'||}:
http://math.stackexchange.com/questions/70509/can-anyone-tell-me-why-the-arclength-integral-is-a-lower-semicontinuous-function
Аноним 07/06/16 Втр 21:51:40 #451 №367893 
>>367857
>Edit, 12/10/15: An earlier version of this answer claimed that no such map can be smooth. In fact this is false; a counterexample can be constructed by slowing down as you hit each corner using a bump function
Вот это уже ближе к делу. Но опять же не достаточно для ответа (как он именно применяет сглаживание). Хотя Киачу и заслуживает доверия.

Аноним 07/06/16 Втр 21:56:45 #452 №367894 
>>367764
Еще вопрос кароч. Рассмотрим топологию порядка на расширенной вещественной прямой. Рассмотрим, например, функцию f(x) = 1/x^2. Доопределим ее: пусть f(0) = ∞. Тогда получается, что f непрерывна, если ее область значений имеет описанную выше топологию. Теперь расширим область определения f и положим f(-∞) = f(∞) = 0. Тогда f все еще непрерывна. Я правильно понимаю?
Аноним 07/06/16 Втр 21:57:28 #453 №367895 
>>367894
Да.
Аноним 07/06/16 Втр 21:58:20 #454 №367896 
>>367894
Нет, вернее. Поясни что такое расширенная вещественная плоскость.
Аноним 07/06/16 Втр 22:03:33 #455 №367897 
>>367573
еб вашу мать, мне про это кто-нибудь блять ответит гавно ебаное сука нахуй !?
Аноним 07/06/16 Втр 22:07:36 #456 №367898 
>>367897
Курс очень милый. Стоит.
Аноним 07/06/16 Втр 22:09:55 #457 №367899 
>>367896
Не плоскость а прямая. Расширенная вещественная прямая R' - это R и еще +- ∞. Операции определены по очевидным правилам (∞ - ∞ не определено; есть неочевидное правило: 0∞ = 0). Обычно так не делают, потому что R' теряет алгебраические свойства R. Это не поле и есть пары элементов, для которых даже сложение не определено. Но пацаны пишут, что в теории меры так делать удобно. В качестве топологии я беру топологию порядка.
Аноним 07/06/16 Втр 22:13:33 #458 №367900 
>>367899
Это я писал. Так делать удобно только на полупрямой [0..inf], а не на прямой [-inf..inf].

1/x^2 не непрерывно на [-inf..inf] потому что непонятно, куда отправлять 0 (в inf или -inf?)

Аноним 07/06/16 Втр 22:19:09 #459 №367902 
>>367897
Очевидно же, что да, стоит. Каледин вообще редко пишет хуйню.
Аноним 07/06/16 Втр 22:19:14 #460 №367903 
>>367900
В inf. Я специально взял 1/x^2 а не 1/x.
Аноним 07/06/16 Втр 22:21:18 #461 №367904 
>>367903
Нет не непрерывна.
Аноним 07/06/16 Втр 22:23:31 #462 №367906 
Для чего нужны ряды? Для чего их применяют, я что то и не понял пока.
Аноним 07/06/16 Втр 22:23:41 #463 №367907 
>>367899
На самом деле так делают. Рудин делает.
Аноним 07/06/16 Втр 22:32:17 #464 №367910 
>>367907
Страницу где он определяет 0 inf = 0.
Аноним 07/06/16 Втр 22:43:17 #465 №367911 
>>367904
Почему?
Аноним 07/06/16 Втр 22:43:34 #466 №367912 
>>367904
>>367911
В смысле покажи разрыв.
Аноним 07/06/16 Втр 22:49:29 #467 №367913 
>>367911
Прообраз (открытого) полуинтервала (1 +inf] будет [0..1) - не открытое множество.
Аноним 07/06/16 Втр 22:52:12 #468 №367914 
>>367913
Еще раз: функция 1/x^2, а не 1/x. Прообраз (1; +inf] будет (-1, 1).
Аноним 07/06/16 Втр 22:56:25 #469 №367915 
>>367914
Да, ты прав. Тогда будет.
Аноним 07/06/16 Втр 23:01:00 #470 №367918 
Здесь есть кто-нибудь любитель теории типов, гомотоп. теории типов и катов с упором на пруф ассистанты?
Аноним 07/06/16 Втр 23:29:18 #471 №367923 
>>367893
Замечание только, что это не совсем тот вопрос. Это не про диффеоморфность окружности и квадратика вообще, а как подмногообразий в R^2.
Аноним 08/06/16 Срд 11:00:13 #472 №367990 
https://www.youtube.com/watch?v=vHj9_2ObydA
Топология дает возможность ВСЕМ пройти к успеху.
Тополооооогия дает возможность всем пройти к успеху, реально.
Топология - это тема. Топология -
это
наша
тема.
Аноним 08/06/16 Срд 12:32:14 #473 №368016 
>>367914
Еще интересная тема про эту топологию (почему она плохая). Если f, g: X -> R' непрерывны, их произведение fg не обязано быть непрерывным:
Пусть, например f(x) = 1/x^2 при x != 0 и inf при x = 0, g(x) = x^2. Тогда обе эти функции непрерывны в описанной выше топологии, но fg = 1 при x != 0 и 0 при x = 0. Короче, юзлесс топология, не буду ее использовать.
Аноним 08/06/16 Срд 12:52:30 #474 №368025 
>>368016
Так что я про эту топологию забуду и не буду вспоминать никогда. Жалко, что вообще думал про нее несколько часов.
Аноним 08/06/16 Срд 13:13:25 #475 №368032 
>>367990
Тпология это йоба, а YOBA ETO TI
Аноним 08/06/16 Срд 17:17:06 #476 №368145 
14653954270240.png
14653954270251.png
14653954270262.png
14653954270263.png
Упражнение 2а должно быть таким сложным или я туплю и оно на самом деле проще делается? В решении я использовал упражнение 1 (очень простое) и тот факт, что произведение измеримых функций в R измеримо.

R с чертой означает R и +- inf. BX означает борелевскую сигма-алгебру на X. Измеримость на подмножестве определяется на пикрелейтед 3. Борелевская сигма-алгебра на R с чертой определяется на пикрелейтед 4.

У меня 2 вопроса: есть ли более простое решение и какого хуя такое, казалось бы, простое утверждение так сложно доказывается? До этого я еще полдня пытался ввести топологию на R с чертой так, чтобы она порождала сигма-алгебру на пикрелейтед 4 и доказать это утверждение для R с чертой точно так же как его доказывают для R, обрадовался, когда ввел, а потом оказалось, что в этой топологии умножение не является непрерывной операцией.
sageАноним 08/06/16 Срд 17:24:44 #477 №368146 
14653958845200.png
Вглядитесь в эти лица. Это посетители имиджборд. Они уроды.
А теперь посмотри на себя. Замечаешь сходство? То-то и оно.
Аноним 08/06/16 Срд 17:41:36 #478 №368149 
ХЗ где лучше, спрошу здесь. Что-то я туплю аноны. Хочу посчитать RMS синуса числовым методом.
На вскидку можете сказать, по скольким точкам минимум нужно считать (порядок), чтобы получилось близко к 1/sqrt(2), хотя бы до 3-го знака 0.707?

Написал прогу, которая считает всего по 10 точкам. Получил на удивление точный результат:

1: deg 0.0, rad 0.0
2: deg 36.0, rad 0.6283185307179586
3: deg 72.0, rad 1.2566370614359172
4: deg 108.0, rad 1.8849555921538759
5: deg 144.0, rad 2.5132741228718345
6: deg 180.0, rad 3.141592653589793
7: deg 216.0, rad 3.7699111843077517
8: deg 252.0, rad 4.39822971502571
9: deg 288.0, rad 5.026548245743669
10: deg 324.0, rad 5.654866776461628

nsteps: 10, rms: 0.7071067811865476

Но как видите не хватает последней точки 2PI, то есть интервал получился от 0 до 18pi/10. Если ее добавить, получим в ней нуль, но кол-во шагов будет 11, и если разделим на 11 то получим 0.67, что даже неблизко. ОДНАКО! Если увеличивать кол-во точек, то 0.67 начнет приближаться к 0.707 где-то после 10000 шагов. Что в принципе имеет смысл - чем больше итераций, тем точнее. Но почему я же тогда в 1-м случае я получил такой точный результат за 10 шагов, на интервале от 0 до 18pi/10? Это читинг?
Аноним 08/06/16 Срд 18:17:44 #479 №368152 
Планирую за лето качественно изучить весь материал, предложенный в оп-посте, и стать самым гениальным человеком всех времён. Какие подводные камни?
Аноним 08/06/16 Срд 18:22:08 #480 №368153 
>>368152
На изучение одной теоремы придётся тратить два-три дня.
Аноним 08/06/16 Срд 19:21:29 #481 №368160 
>>368152
Средняя скорость чтения математической литературы - 1 страница в час.
Аноним 08/06/16 Срд 19:24:45 #482 №368161 
>>368160
Причем без учета упражнений.
Аноним 08/06/16 Срд 19:32:18 #483 №368163 
>>368160
Да ладно, это слишком медленно. Я вот первый том Зорича на english сейчас читаю. Примерно пять минут-семь на страницу. Разве дальше должно стать настолько
>час на 1 страницу
хуже?
Аноним 08/06/16 Срд 19:37:24 #484 №368164 
>>366963 (OP)
Анон, чему равна мера Жордана множества N?
Что-то я запутался. С одной стороны, нижняя мера равна 0, верхняя inf, а потому множество неизмеримо, с другой же — граница N состоит только из одной точки, потому мера границы равна нулю и множество измеримо. Где ошибка, помоги плз.
Аноним 08/06/16 Срд 19:47:12 #485 №368167 
>>368164
Не из одной точки. Граница N есть N. Внутренность N - пустое множество.
Аноним 08/06/16 Срд 19:56:05 #486 №368170 
>>368167
> Граница N есть N
Хм, почему? Это следует из того, что окрестность точки n не проколотая, потому пересечение с N содержит точку n?
Аноним 08/06/16 Срд 20:14:53 #487 №368171 
>>368170
Внутренность множества M - множество таких точек x∈M, что существует открытое множество U такое, что x∈U и U⊂M. Внутренность M обозначается как IntM.

Граница множества M - множество M\IntM. То есть граница M - множество таких точек x∈M, что x∉IntM. Граница M обозначается BnM.

В стандартной топологии вещественной прямой не существует такого непустого открытого множества U, что U⊂ℕ. Поэтому Intℕ = ∅. Поэтому Bnℕ = ℕ\∅ = ℕ.
Аноним 08/06/16 Срд 20:31:32 #488 №368174 
>>368171
>граница M - множество таких точек x∈M
Не обязательно. Границей открытого интервала (a;b) есть множество {a;b}, хотя ни а, ни b интервалу не принадлежат.
Я отталкивался от определения, данного в википедии, где граничной точкой множества А называется точка, окрестность которой имеет непустое пересечение как с самим множеством А, так и с его дополнением. То есть для x найдется окрестность U (не проколотая) такая что
U ∩ A ≠ ∅ и U ∩ A^c ≠ ∅, при чем сама точка х не обязана принадлежать множеству А.
В таком случае я согласен, что граница N есть N, так как окрестность U содержит точку х и некоторые точки, не являющиеся элементами N, потому любое натуральное число есть граничная точка N.

Но все же, что это говорит об измеримости N? Оно не измеримо?
Аноним 08/06/16 Срд 20:33:02 #489 №368175 
>>368163
Ну если эти концепции тебе более-менее знакомы, то понятно, что быстро читаешь. Я могу вообще за 20-30 секунд страницу просмотреть, если там для меня ничего нового. А если что-то новое, то и больше часа можно думать. Не потому что какую-то строчку не понял, а просто надо самому себе задавать вопросы и ставить задачи во время чтения.
Аноним 08/06/16 Срд 20:35:36 #490 №368176 
Пусть у нас есть функции f: X -> Y, g: X -> Z.
Кроме того, f(x) = g(x) для всех x. Равны ли эти функции? По идее функции - это подмножества декартовых произведений. Могут ли подмножества X x Y и X x Z быть равны, если Y != Z?
Аноним 08/06/16 Срд 20:40:04 #491 №368177 
>>368176
Если f(x) = g(x) для всех x, то Im(f)=Im(g), области значений функций равны как множества. Вообще говоря, Im(f)≠ Y, Im(g)≠ Z, они лишь подмножества этих множеств (которые могут и совпадать с ними, но необязательно). То есть главное, чтобы области значений совпадали и условие "f(x) = g(x) для всех x" выполнялось. Обычно равенство функций так и задается имхо
Аноним 08/06/16 Срд 20:44:39 #492 №368179 
>>368174
N измеримо, его мера равна нулю, так как его внутренность пуста. такой-то буддизм
Аноним 08/06/16 Срд 20:45:40 #493 №368180 
>>368179
Верно ли тогда, что любое счетное множество или имеет меру нуль, или неизмеримо?
Аноним 08/06/16 Срд 20:51:04 #494 №368181 
>>368176
Для любого элемента x∈X существует единственный такой элемент y∈Y, что пара (x,y) ∈ f. Обозначается y = f(x).

Для любого элемента x∈X существует единственный такой элемент z∈Z, что пара (x,z) ∈ g. Обозначается z = g(x).

Предположим, что f != g. Тогда существует такая пара (x',p), которая является элементом одной из этих функций, но не является элементом другой. Но это невозможно, так как из условия следует, что f(x') = g(x').
Аноним 08/06/16 Срд 20:52:06 #495 №368182 
>>368180
Любое счетное множество имеет меру нуль.
Аноним 08/06/16 Срд 20:54:09 #496 №368183 
>>368182
Но как же классический пример множества рациональных чисел из отрезка [0;1], которое неизмеримо по Жордану?
Аноним 08/06/16 Срд 21:00:10 #497 №368184 
>>368183
Все счётные множества измеримы по Лебегу. Мера Жордана равна мере Лебега, если она есть.
Аноним 08/06/16 Срд 21:05:28 #498 №368185 
>>368184
>Ограниченное множество измеримо по Жордану тогда и только тогда, когда его граница имеет меру Жордана нуль (или, что равносильно, когда его граница имеет меру Лебега нуль).
Граница множества рациональных чисел из отрезка [0;1] очевидно, это множество как ограничено, так и счетно равна 1, из чего следует, что это множество неизмеримо по Жордану. Также это следует из того, что верхняя мера этого множества равна 1, а нижняя 0, потому оно неизмеримо. Если ты говоришь, что любое счетное множество измеримо, то вот тебе пример неизмеримого счетного множества.
или мы говорим о разном, лол?
Аноним 08/06/16 Срд 21:11:03 #499 №368187 
>>368185
>Граница множества рациональных чисел из отрезка [0;1] очевидно, это множество как ограничено, так и счетно равна 1
Мера границы.
фикс
Аноним 08/06/16 Срд 21:13:31 #500 №368188 
>>368185
Если множество измеримо по Жордану, то оно измеримо по Лебегу, и меры равны.
Любое счётное множество измеримо по Лебегу.
Мера Лебега счётного множества равна нулю.

Поэтому если счётное множество измеримо по Жордану, то оно имеет меру нуль.
Аноним 08/06/16 Срд 21:16:15 #501 №368189 
>>368188
>если счётное множество измеримо по Жордану
Согласен, но из этого не следует, что любое счетное множество измеримо по Жордану. Те, что измеримы, имеют меру нуль, это я уже понял, спасибо.
Аноним 08/06/16 Срд 21:19:50 #502 №368191 
>>367906
Я сам нуб и многое забыл, но вспомнить хотя бы ряд Тейлора / Лорана -- разложили функцию в ряд и нашли приближенное значение в окрестности точки.
Аноним 08/06/16 Срд 21:35:02 #503 №368192 
>>368189
Ну да. не следует. В >>368182 говорилось про Лебега.
Аноним 08/06/16 Срд 21:46:13 #504 №368195 
>>368149
Ау бурбаки, пособите, ну. Кроме гамологий в че-нить можете.
Аноним 08/06/16 Срд 21:56:06 #505 №368197 
>>368195
Потому что точка 2пи сумму не меняет, зато числитель уменьшается в силу увеличения знаменателя, ты сам об этом сказал.

В чем вообще основная задумка-то? Сумма в числителе сходится, или что?
Аноним 08/06/16 Срд 22:08:21 #506 №368199 
>>368197
Просто не понимаю почему без включения 2пи то есть типа на [0, 2pi[ сходится к 1/sqrt(2) мгновенно при достаточно грубой дискретизации - это подозрительно. Если же делать "честно" [0, 2pi] то требуется несколько тысяч итераций. Думаю может ошибка в программе? Думал может какой теоретический инсайт мне дадите, уровня duh.
Аноним 08/06/16 Срд 22:10:41 #507 №368200 
Не знаю уместно тут код постить. Вот так работает заебись. Стоит сделать i <= 360, и уже нужно не 11 шагов а тыщи их, чтобы сходилось к 0.707 хотя бы.

public class Rms2
{
public static void main (String[] args)
{
double step = 36.;
double sum = 0.;
int nsteps = 0;
for (double i = 0; i < 360; i += step)
{
double r = i Math.PI / 180.;
System.out.println ((nsteps+1) + ": deg " + i + ", rad " + r);
sum += Math.sin(r)
Math.sin(r);
++nsteps;
}

double rms = Math.sqrt(sum/nsteps);

System.out.println ("\nnsteps: " + nsteps + ", rms: " + rms);
}
}
Аноним 08/06/16 Срд 22:12:21 #508 №368202 
>>368199
> Если же делать "честно" [0, 2pi] то требуется несколько тысяч итераций
Ну вот у тебя 11 член суммы это квадрат синуса 2пи, насколько я понял? Из какого интервала остальные тысячи? Возможно, ты каждый раз захватываешь точку 2kpi, каждая из которых ухудшает результат. Исключи из повторений значения 2kpi и посмотри, что получится.
Аноним 08/06/16 Срд 22:17:20 #509 №368203 
>>368202
Два варианта

1) На интервале [0, 2pi[ делим интервал на 10 частей и это достаточно для получения супер точного результата

2) На интервале [0, 2pi] для получений похожей точности внезапно требуется дельту уменьшить в 1000 раз.

Аноним 08/06/16 Срд 22:19:20 #510 №368204 
Вот так без табов надеюсь. Вот это 10 шагов.
Если сделать i=< 360, то есть включаем 2pi, получаем 11 шагов и результат 0.6??.


public class Rms2
{
public static void main (String[] args)
{
double step = 36.;
double sum = 0.;
int nsteps = 0;
for (double i = 0; i < 360; i += step)
{
double r = i Math.PI / 180.;
System.out.println ((nsteps+1) + ": deg " + i + ", rad " + r);
sum += Math.sin(r)
Math.sin(r);
++nsteps;
}

double rms = Math.sqrt(sum/nsteps);

System.out.println ("\nnsteps: " + nsteps + ", rms: " + rms);
}
}
Аноним 08/06/16 Срд 22:21:24 #511 №368205 
>>368203
Исключи точки, в которых синус равен нулю 2kpi, они только ухудшают результат. В первом случае попробуй исключить еще и ноль.
Аноним 08/06/16 Срд 22:24:55 #512 №368206 
>>368205
>2kpi
Бля, kpi просто.
Аноним 08/06/16 Срд 22:26:17 #513 №368207 
>>368205
Да, я наверную могу подfudgить чуток, но мне просто интересно - теоретически RMS синуса равно 1/sqrt(2) на интервале [0, 2pi], так?
То есть мы не должныев принципе ничего выкидывать а честно подставлять 0<=360 в градусах. И ведь в конечном итоге получается правильный результат - только требуется существенно увеличивать частоту дискретизации. Что в принципе имеет смысл.
Так что я склоняюсь что мой результат с выколотой 2pi это какой lucky coincidence который наверное можно объяснить, но я не знаю как.
Аноним 08/06/16 Срд 22:31:26 #514 №368208 
Я просто хочу запилить true RMS вольт метер. У мамки на даче форма напряжения - просто пиздец. Синусом это трудно назвать. Мне правда ебнуло хорошенько, пока я скопом мерял, но это неважно. Она подозревает, что счетчик ее наебывает. Я подумал - такое может быть, если счетчик аналоговый и откалиброван тупо умножать пик на 0.707. Но если синус говно, там прут всякие гармноки, короче тру RMS может быть существенно меньше, а так как платим за мощность - может быть ошибка. Я хочу сделать MK + ADC, мерять за период и вычислять тру RMS и с этим уже идти в мухэнерго. И для меня естественно важно по 10 точкам считать или по 10 тысячам. Хотелось бы по 10 естеснно.
Аноним 08/06/16 Срд 22:36:08 #515 №368209 
>>368207
>RMS синуса равно 1/sqrt(2) на интервале [0, 2pi]
Хуй знает, пруф есть? Просто не совсем представляю предельный переход в данном случае.
Если ты про среднее квадратическое, то его значение в твоем случае зависит не только от итераций, но и от разбиения.
Аноним 08/06/16 Срд 22:37:03 #516 №368210 
>>368208
> true RMS
У него же формула не такая, как у обычного RMS, не?
Аноним 08/06/16 Срд 22:42:51 #517 №368211 
>>368210
Такая же. Вытекает из необходимости высчитывать эквивалент потребляемой мощность для DC на интервале t. P(t) = I(t)^2 x R.
И если вместо I подставить интеграл, или сумму, то как раз получится RMS. Прикол в том, что многие долбоебы объясняют необходимость использовать RMS "потому что синус меняет знак" - то есть якобы это блять такой математический трюк. Это у статистов трюки и форексов. А тут матушка природа.
Аноним 08/06/16 Срд 23:05:26 #518 №368216 
14654163261850.jpg
>>368209

Хм, пруф не задумывался, но вот интеграл посчитал вроде все сходится. sin^2 преобразовал через power reduction и под интегралом остается 1-cos , после интегрирования 1-sin и подстановки пределов sin=0, короче получается 1/2. wicked.
Аноним 08/06/16 Срд 23:33:25 #519 №368226 
Здорово, аноны. Какое сейчас наиболее перспективное и быстро развивающееся направление математики?
Аноним 09/06/16 Чтв 00:25:54 #520 №368238 
>>368226
Если обобщить, алгеом и HoTT.
Аноним 09/06/16 Чтв 00:29:36 #521 №368239 
>>368238
>hott
Пхаха
Аноним 09/06/16 Чтв 00:32:40 #522 №368241 
>>368226
В математике нет направлений.
Аноним 09/06/16 Чтв 08:43:43 #523 №368268 
>>367863
Анализ 1го курса, пиздец пакет терминов, ya ebal.
Аноним 09/06/16 Чтв 09:39:33 #524 №368273 
>>367079
>После этого начинать Винберга, Рудина, Кострикина-Манина, это будет как удар лбом об стену
Нет. После миллионов вычислительных задач, отличающихся только кол-вом чисел и правилами, как их между собой скрещивать; Винберг, Рудин и Костркин-Манин будут манной небесной.
Аноним 09/06/16 Чтв 14:46:07 #525 №368312 
Алгебру Бурбаки можно читать для первого обучения этой области?
Хорошо разбираюсь в теории множеств.
Аноним 09/06/16 Чтв 15:08:32 #526 №368314 
14654741127420.png
Примат из полушараги ворвался.
Как выучить семестровый курс за 1.5 дня? Экзамен в субботу.
Аноним 09/06/16 Чтв 15:14:19 #527 №368316 
>>368314
Никак, только списать.
Аноним 09/06/16 Чтв 15:21:10 #528 №368317 
>>368312
da
Аноним 09/06/16 Чтв 15:26:27 #529 №368318 
>>368314
На троечку можно подучить, если весь семестр не пинал хуи.
Аноним 09/06/16 Чтв 15:42:36 #530 №368320 
>>368314
А почему ты пинал хуи весь семестр?
Аноним 09/06/16 Чтв 16:19:33 #531 №368326 
>>368316
Ну там теорию прям перед преподом садишься писать, даже не знаю...
>>368318
Пинал как раз.
>>368320
Не знаю, даун видимо. Поначалу прям учусь целую неделю, а потом как-то хуй забиваю и всё тут.

Вообще в первом семестре тоже нихуя не делал и прокатило, но вроде и материал был полегче...
Буду Аллаху, молиться, мда уж.
Аноним 09/06/16 Чтв 16:59:23 #532 №368336 
А когда вы шли на математику, то какой примерно имели уровень знаний? Есть те, кто с довольно слабым уровнем выжил?
Аноним 09/06/16 Чтв 16:59:24 #533 №368337 
>>368314
31 вопрос всего, можно успеть подучить
Аноним 09/06/16 Чтв 17:00:12 #534 №368338 
>>368337
Если там ещё доказательства надо писать, то надорвётся.
Аноним 09/06/16 Чтв 17:02:18 #535 №368339 
>>368336
Я два семестра не верил, что между двумя отрезками разной длины может быть биекция, и срался с преподом из-за этого.
Аноним 09/06/16 Чтв 17:07:21 #536 №368341 
>>368336
Да. Школьная математика нахуй не нужна, разве только при сдаче егэ. Физматовцы только в начале тащат, потом курсу к 2 у всех примерно один и тот же уровень.
Аноним 09/06/16 Чтв 19:14:59 #537 №368351 
>>368341
Утешил.
Я вообще на механику идти собираюсь, но первые два курса у них программа идентичная с математиками.
Аноним 09/06/16 Чтв 19:26:50 #538 №368356 
>>368341
Смерти тебе.
Аноним 09/06/16 Чтв 20:06:05 #539 №368364 
>>368336
У меня был такой себе. Ничего кроме школьной не знал, гомологии как Вербила в 9ом классе не считал. Ещё рано говорить, выжил или нет, но статейки какие-то есть, правда в ожидании рецензии, но такое.
Аноним 09/06/16 Чтв 20:30:59 #540 №368368 
>>368364
Какой курс?
Аноним 09/06/16 Чтв 20:32:07 #541 №368369 
>>368368
3
Аноним 09/06/16 Чтв 20:33:32 #542 №368370 
Анон, при делении натурального числа n на простое число p (при условии, что n не кратно р) мы получаем бесконечную периодическую дробь. Как это доказать, хз. Но вот примеры:
1/3=0,(3)
1/7=0,(142857)
1/11=0,(09)
1/13=0,(076923)
Дроби 1/2, 1/5 не образуют периодов наверняка как-то связано с тем, что 2 и 5 есть делители 10 И так далее.
При умножении получившихся дробей на другие натуральные числа получаются дроби с другими периодами. Но, при умножении дроби 1/7 на n она не меняет период, а лишь терпит некоторые перестановки. Например: 2/7=0,(285714)
3/7=0,(428571)
То есть, можно сказать, что в некотором смысле период этой дроби образует группу относительно операции перестановок. Может ли кто-то объяснить, почему деление на 7 так примечательно? Или это вообще школьная истина, а я долблюсь в глаза?
Аноним 09/06/16 Чтв 20:35:18 #543 №368371 
>>368216
>power reduction
Kek
Аноним 09/06/16 Чтв 20:40:15 #544 №368373 
>>368239
Что не так?
Аноним 09/06/16 Чтв 21:46:49 #545 №368395 
>>368371
чому кек, как правильно
Аноним 09/06/16 Чтв 22:50:16 #546 №368486 
>>368370
Короче хз как. Но любая дробь это бесконечная дробь, то есть 1 = 1.(0). Докажи, что произведение периодических дробей есть периодическая дробь. Потом если дробь непериодическая, то дробь нерациональная. Может я неправ, но это сходу.
Аноним 09/06/16 Чтв 22:56:07 #547 №368491 
>>368486
> Докажи, что произведение периодических дробей есть периодическая дробь
Это уже доказано, если вводить вещественные числа как бесконечные дроби.
>Потом если дробь непериодическая, то дробь нерациональная.
В любом случае они периодические, тк числа вида n/p рациональны. Но для других р период зависит от числителя n. Вопрос в том, почему при делении на 7 период не меняется, а при делении на другие простые числа — меняется.
Аноним 09/06/16 Чтв 22:58:13 #548 №368493 
>>367429
Ап вопроса.

Нашел вот такое

https://books.google.ru/books?id=KxBCpzyNi_IC&lpg=PA22&ots=_-TKcyPF6b&dq=grassmannian%20transition%20charts&pg=PA22#v=onepage&q&f=false

Но тут чот без поллитры не разберешься.
Никто не встречал обьяснения попроще, для конкретной размерности?

Аноним 09/06/16 Чтв 22:59:46 #549 №368494 
>>368491
Может там что-то написано будет https://en.wikipedia.org/wiki/Repeating_decimal#Fractions_with_prime_denominators
Аноним 09/06/16 Чтв 23:02:44 #550 №368495 
>>368494
Спасибо, тут нашел все:
https://en.wikipedia.org/wiki/Cyclic_number
Аноним 10/06/16 Птн 03:32:27 #551 №368521 
Аноны, стоит листки в список для самых маленьких включать? Киньте ссыль на них, плз.
Аноним 10/06/16 Птн 05:27:09 #552 №368527 
>>367153
yasobe.ru/na/flores
Аноним 10/06/16 Птн 05:45:43 #553 №368530 
>>368521
http://ium.mccme.ru/old_courses.html
Аноним 10/06/16 Птн 16:26:16 #554 №368669 
>>368530
Ебанарот, это не для самых маленьких.
Аноним 10/06/16 Птн 16:31:38 #555 №368673 
Анон, как построить многочлен исходя из множества его частных значений? И можно ли вообще?
То есть, у меня есть конечное множество {x(n)} и множество {f(x(n))}.
Аноним 10/06/16 Птн 17:05:42 #556 №368685 
>>368673
Многочлены Лагранжа, Ньютона.
Аноним 10/06/16 Птн 17:09:09 #557 №368687 
>>368685
Спасибо.
Аноним 10/06/16 Птн 20:14:15 #558 №368723 
Аноны, помогите. Надо выучить теорию случайных процессов за три дня. Есть годная литра на эту тему, написанная простым языком?
Аноним 10/06/16 Птн 21:38:49 #559 №368745 
>>366963 (OP)
Нука накидали что про вариационное исчисление
Аноним 10/06/16 Птн 23:20:05 #560 №368773 
>>366963 (OP)
Почему математика так важна?
Аноним 11/06/16 Суб 00:22:14 #561 №368781 
>>368204
АААА, понял какой я еблан, просто ж нужно как обычный интеграл кодить например через mid point считать, а я тупо суммирую f(x) не умножая на ∆x!!!!

Меня смутило что, когда про площадь я думаю, то очевидно что кодить надо ∑ f(x)∆x.

А вот про среднее у меня брейнфарт приключился, ведь в дискретном виде сумма ∑ai, то есть просто суммируем значения. Вот я и просуммировал сука. А для непрерывной функции среднее эта та же площадь, деленная на интервал. Нужно было в программерском форуме спрашивать, здесь похуй на прикладные дела, здесь же N определяют.

Аноним 11/06/16 Суб 19:42:35 #562 №368916 
Короче я тут спрашивал как выучить матан...
В общем на 3 сдал, нормас. 3 вопроса сам написал, а 2 кое-как скатал. Вооот.
просто поделилися
Аноним 11/06/16 Суб 21:33:13 #563 №368953 
>>368916
Молодец.
Аноним 12/06/16 Вск 04:49:50 #564 №368976 
Помните в тории вероятность есть какой-то парадокс про выбор двери? А помните ещё задачу про вероятность что корова встанет? Так вот я придумал как их объединить. Корова с средним спит 3 часа к примеру. Как посчитать вероятность того что корова проснётся в 6 часов? Ведь чем больше корова спит тем больше вероятность что она проснётся. Значит эта вероятность не большая т.к. вероятность того что корова проснётся раньше гораздо больше. А если известно что корова уже спит 2 часа? Абстрагируемся от коров и перейдём на двери. Есть 7 дверей. Номер каждой из них означает через сколько часов проснётся корова и 1 означает больше 6. Чтобы посчитать среднее время сна коровы провели эксперимент. Замерили время сна m коров и x[j] это количество коров проснувшихся в j-ом часу. Значит вероятность проснуться в n-ом часу равно f(n)=x[n]/m. Больше всего она в 3 и по гаусовкому распределению равномерно уменьшается к краям. Если мы хотим угадать где корова, то логично выбрать что она за 3-ей дверью. Вероятность ошибки равна 1-f(3). Теперь открыты первые 2 двери и там коровы нет. Пусть вероятность что корова была за одной из открытых равна k=f(1)+f(2). Тогда вероятность ошибки теперь g(n)=1-f(n)-k. Значит шанс выигрыша других дверей увеличились, а третей остались 1-f(3). Пусть номер двери с максимальным шаносом равен max. В данном случае 3. Теперь открыли несколько дверей и, коровы там не оказалось и вероятность что за ними была бы корова равна k. Тогда если есть такое n при котором f(max) < f(n)+k, то лучше выбрать n чем max. Вот вам и парадокс. Что скажите?
sageАноним 12/06/16 Вск 19:34:36 #565 №369028 
>>368146
Да иди ты нахуй со своим сходством.
Аноним 12/06/16 Вск 21:05:06 #566 №369035 
>>369028
удваиваю. хуйня какая-то
Аноним 12/06/16 Вск 23:01:18 #567 №369051 
14657616791230.gif
Короче, тут сверху проснулись насчёт физики и математики и реальности в математике. И натыкаюсь на такую статью в вики. https://ru.m.wikipedia.org/wiki/Математическая_физика Что за хуйня?
Sage Аноним 12/06/16 Вск 23:01:57 #568 №369052 
>>369051
*просрались
Аноним 13/06/16 Пнд 01:04:34 #569 №369068 
П-П-П-П-П-П-ПЕРЕКОТ

>>369067 (OP)
comments powered by Disqus

Отзывы и предложения